You are on page 1of 66

SỞ GIÁO DỤC VÀ ĐÀO TẠO KỲ THI CHỌN HỌC SINH GIỎI

CẤP TỈNH LỚP 9 NĂM 2018


MÔN: TIẾNG ANH
(Đề thi gồm có 08 trang) Thời gian làm bài: 120 phút (không kể thời gian phát đề)
Ngày thi: …………………….

ĐỀ 1
SECTION I: LISTENING (3.0 points) (You will hear twice for each part)

Part 1. You will hear a doctor talking about how people can lead a healthier life. For each
question, put a tick in the correct box.
1. To become healthier you should _______.
A. dramatically change your life
B. change some daily habits.
C. eat hardly anything.
2. If you don't manage to exercise as much as you should _______.
A. leave the gym.
B. try not to be negative about it.
C. be angry with yourself.
3. To improve your mood you should ________.
A. drink more tea and coffee.
B. only eat vegetables.
C. increase the amount of vegetables you eat.
4. The survey ________.
A. showed quite dramatic results.
B. didn't have strong results.
C. didn't give any useful information.
5. You should always ________.
A. do important jobs first.
B. do everything as quickly as possible.
C. fry to finish what you start
6. The doctor says ________.
A. you should never have a late night
B. lack of sleep causes brain disease.
C. it's okay to go to bed late sometimes.

Part 2. You will hear a tour guide giving information about an old British house. For each
question. fill in the missing information in the numbered space.
- The house was built in the (7) ……………………..………..
- The Reynold family lived in the house until (8) ……………………..………..
- The servants had rooms in the (9) ……………………..………..
- The art collection is in the (10) ……………………..………..
- George Reynold was a (11) ……………………..………..
- George's son Michael died from (12) ……………………..………..
- Wife died while giving birth to their (13) ……………………..………..Emily.
- George's brother died in a (14) ……………………..………..riding accident.
- George died of a heart attack at the age of (15) ……………………..………..

SECTION II: LEXICO – GRAMMAR (7.0 points)


Part 1: Questions from 16 to 36. (3.0 points - 0.15/ each)
Choose the best answer to each question.
16. As a citizen, it is normal to _________ to the laws and rules made by the society.
A. conform B. resist C. hinder D. obey
17. He attempted to _________the painting from the gallery but he was caught.
A. rob B. steal C. thief D. kidnap
18. Once the computer virus was removed, a lot of information ________.
A. is disappearing B. will have disappeared
C. disappears D. disappeared
19. Telecommuter boosters will often mention _________convenient it is for people wishing
to cut down on their work hours.
A. how B. therefore C. only D. most
20. A: “Will the Jackson be invited?”– B: “I_________.”
A. expect so not B. so expect C. expect so D. don’t expect
21. I read the contract again and again ___________avoiding making spelling mistakes.
A. with a view toB. on account of C. by means of D. in terms of
22. In a report submitted to the government yesterday, scientists __________that the
building of the bridge be stopped.
A. banned B. complained C. saidD. recommended
23. Anyone ordering a new MP3 player _________ the end of the month will receive a free
extra set of headphones.
A. at B. in C. upon D. before
24. I enjoyed reading that story. It was rather sad, _________.
A. also B. though C. but D. so
25. My job is so ________ that I don’t think I’ll be able to take a summer break.
A. persisting B. hard C. demanding D. tough
26. A number of students ________ for a rise since last year
A. asked B. has asked C. ask D. have asked
27. I have been fascinated by _________ since I was at secondary school.
A. photographic B. photographer C. photograph D. photography
28. She has difficulty ________ and ________ English.
A. speak-writing B. to speak- to write C. speak-write D. speaking-writing
29. Her mother’s illness cast a cloud__________her wedding day
A. for B. over C. on D. in
30. I don’t really___________winter sports very much
A. deal with B. face up to C. get round to D. go in for
31. The meeting was _______ off at the last minute because the papers were not ready.
A. stopped B. broken C. called D. turned
32. There are still, however, rather a lot of obstacles to ________.
A. overcome B. come over C. solve D. resolve
33. __________ the barrier at the side of the road, the car would have crashed into the valley
below.
A. But for B. Apart from C. Except for D. Unless
34. She has quite a good _____ of English.
A. result B. pass C. understanding D. head
35. According to the _____ of the contract, tenants must give six months notice if they intend
to leave.
A. laws B. rules C. terms D. details

Part 2: Questions from 36 to 45. (2 points - 0.2/ each)


Use the correct form of the word in brackets to complete the following sentences.
36. There are some ……………………..………..to every grammatical rule. EXCEPT
37. The engineering sector achieved significant ……………………..………..last year. GROW
38. I have a ……………………..………..of old records from my grandma. COLLECT
39. He'll ……………………..………..be coming later. SURE
40. Eric Clapton's guitar solos are ……………………..………... LEGEND
41. It was one of the most ……………………..………..plays I've seen recently. ENJOY
42. In her time, Marilyn Monroe was a very ……………………..………..actress. GLAMOUR
43. It's ……………………..………..to tell whether he's lying or not. POSSIBLE
44. He's no worldwide star, but he had very humble ……………………..………... BEGIN
45. They took ……………………..………..of noise levels inside the building. MEASURE
Part 3: Questions from 46 to 50. (1 point - 0.2/ each)
Each of the following sentences has an error. Find the errors and correct them.
46. Caroline refused taking the job given to her because the salary was not good.
47. While the brows were away on holiday, their house was broke into.
48. Why don’t you congratulate our son about passing his final exam?
49. There is such few ink that it will be impossible to finish addressing the envelopes.
50. I found it wonderfully to travel abroad.
Part 4: Questions from 51 to 55.(1 point - 0.2/ each)
Fill in each of the sentences with correct prepositions.
51. Have you heard? Sam's got engaged ………..Susan.
52. Let me run ….……..a few of the benefits our scheme offers you.
53. I think that …………..the circumstances it would be a good idea if you stayed at home for a few
days.
54. Therefore, ………..accordance with our regulations, you will be issued with a formal written
warning.
55. Following the financial scandal he left the job ………....disgrace.

SECTION III: READING (6.0 points)


Part 1. Questions 56 – 65 (2 points - 0.2/ each)
Read the passage below and choose the best answer to each of the following blanks.
SOUND ADVICE FOR LANGUAGE LEARNERS
A recent survey of a language learning magazine has consulted a number of experts in
the (56) _______ of a second language acquisition. Their advice may prove invaluable for
those (57) _______ a language course. One suggestion is that you (58) _______ whether you
are likely to be successful at learning a language? Did you enjoy studying languages at school,
for example? The major (59) ________ will be your own time and effort. Therefore, you must
be sure that the course on offer leads to a (60) _______qualification. Also, be realistic in your
(61) ________. If you don’t set achievable aims, you are more likely to give up. Do not be
deceived (62) _______ thinking that the most expensive courses are the best. (63) _________
around to get the best possible value for money. You should also bear in mind that the faster
you learn a language, the more quickly you forget it. Sandra Miller, a French teacher, tried to
teach herself German by rolling on a (64) ________ course. Already fluent in four languages
and with a sound knowledge of teaching methodology her chances of (65) ________ progress
were high. Three years on she remembers very little. She feels her biggest mistake was not to
follow up her first experience. “I should have consolidated what I’d learnt by continuing to
study, even if it were by myself.”
56. A. branch B. field C. area D. domain
57. A. considering B. wondering C. thinking D. looking
58. A. survey B. review C. balance D. assess
59. A. price B. charge C. cost D. valuation
60. A. valued B. regarded C. understood D. recognised
61. A. ends B. sights C. goals D. objects
62. A. into B. about C. by D. in
63. A. Nose B. Shop C. Push D. Run
64. A. rapid B. quick C. fast D. crash
65. A. achieving B. making C. doing D. gaining
Part 2. Questions 66 – 75 (2.0 points - 0.2/ each)
Read the passage below and choose the best answer to each question
The ability to conduct electricity is one of the key properties of a metal. Other solid
material such as silicon can conduct electricity but only effectively at certain temperatures.
Also, some substances such as salt (sodium chloride) can conduct when molten or when
dissolved in water. The ability of metals to conduct electricity is due to how their atoms bond
together. In order to bond together the metal atoms lose at least one of their outermost
electrons. This leaves the metal atoms with a positive charge and they are now strictly ions.
The lost electrons are free to move in what are known as a sea of electrons. Since the
electrons are negatively charged they attract the ions and this is what keeps the structure
together.
An electric current is a flow of charge and since the electrons in the sea of electrons are
free to move they can be made to flow in one direction when a source of electrical energy
such as a battery is connected to the metal. Hence we have an electric current flowing
through the wire, and this is what makes metals such good conductors of electricity. The only
other common solid conducting material that pencil users are likely to encounter is graphite
(what the ‘lead’ of a pencil is made from). Graphite is a form of carbon and again the carbon
atoms bond in such a way that there is a sea of electrons that can be made to flow as an
electric current. Likewise, if we have an ionic substance like salt we can make the electrically
charged ions flow to create a current but only when those ions are free to move, either when
the substance is a liquid or dissolved in water. In its solid state an ionic substance like salt
cannot conduct electricity as its charged ions cannot flow.
Electrical insulators are substances that cannot conduct electricity well either, because
they contain no charged particles or any charged particles they might contain do not flow
easily. Water itself is a poor conductor or electricity as it does not contain a significant
amount of fully charged particles (the ends of a water molecule are partly charged but overall
the molecule is neutral). However, most water we encounter does contain dissolved charged
particles, so it will be more conductive than pure water. Many of the problems that occur
when touching electrical devices with wet hands result from the ever-present salt that is left
on our skin through perspiration and it dissolves in the water to make it more conductive.
66. Electrical conductivity is ________.
A. one of the most important properties of metals
B. one of the key properties of most solid materials
C. impossible for any substance when it is dissolved in water
D. completely impossible for silicon
67. According to the passage, a metal can conduct electricity due to ________.
A. the absence of free electrons B. its atoms with a positive charge
C. the way its atoms bond together D. the loss of one electron in the core of its atoms
68. The word “outermost” in paragraph 1 mostly means ________.
A. the lightest. B. nearest to the inside.
C. furthest from the inside. D. the heaviest.
69. The atoms of a metal can bond together because ________.
A. the lost electrons cannot move freely in the sea of electrons
B. electrons can flow in a single direction
C. they lose all of electrons
D. negatively charged electrons attract positive ions
70. Salt in its solid state is not able to conduct electricity because ________.
A. it has free electrons B. its charged ions can flow easily
C. it cannot create any charge ions D. it charged ions are not free to move
71. The word “they” in paragraph 3 refers to ________.
A. charged ions B. electric currents C. charged particles D. electrical
insulator
72. Water is a poor conductor because it contains ________.
A. no positive or negative electric charge
B. only a small amount of fully charged particles
C. only a positive electric charge
D. only a negative electric charge
73. We can have problems when touching electrical devices with wet hands because
________.
A. the eater itself is a good conductor of electricity
B. the water dissolves the salt on our skin and becomes more conductive
C. the water contains too many neutral molecules
D. the water containing no charged particles makes it more conductive
74. Which of the following is NOT true according to the passage?
A. Pure water is much more conductive than most water we encounter every day.
B. Graphite is a common solid substance that can conduct electricity.
C. Salt can conduct electricity when it is molten or dissolved.
D. Some materials are more conductive than others.
75. Which of the following could best serve as the title of the passage?
A. Electrical Energy B. Electrical Devices C. Electrical Insulators D. Electrical
Conductivity

Part 3: Questions 76 – 85 (2.0 points - 0.2/ each)


Read the text and fill in one word which best fits each gap.
These days it is impossible to open a newspaper (76) …………………..…….. reading about the
damage we are doing to the environment. The earth is being threatened (77) …………………..…….. the
future looks bad (78) …………………..…….. can each of us do? We cannot clean (79) …………………..…….. our
polluted rivers and seas overnight. Nor we can stop the (80) …………………..…….. of plants and animals.
But we can stop adding to the problem (81) …………………..…….. scientists look for answer. It may not
be easy to change your lifestyle (82) …………………..…….. but some steps are easy to take: cut down the
amount of driving you do, or use as little plastic as possible. It is also easy to save (83) …………………..
…….., which also (84) …………………..…….. household bills. We all make a personal decision to work for the
future of our planet if we want to ensure a better world (85) …………………..…….. our grandchildren.

Part 1: Questions from 86 to 90. (2 point- 0.4/ each)


Finish each of the following sentences in such a way that it means the same as the sentence
printed before it.
86. It was careless of you to allow your 16-year-old son to drive your car.
You should
87. I shall never lend John any money, no matter what happens.
Under no
88. “You broke my bicycle, Minh!” said Hoa.
Hoa accused
89. I regret not buying any tickets for the concert.
I wish
90. Tim looks nothing like his father.
Tim doesn’t

Part 2: Questions from 91 to 95. (2 points - 0.4/ each)


Complete the second sentence so that it has a similar meaning to the first sentence, using
the word given. Do not change the word given.
91. I don’t intend to apologize to either of them. (intention)
I have …………………………………………………………………….. to either of them
92. “It was my fault to break your vase yesterday” said Jane to her brother. (admitted)
Jane …………………………………………………………………….. brother’s vase yesterday.
93. You should think about the price before you decide whether to buy it or not.
(consideration)
You should …………………………………………………………………….. before you decide whether to buy it or not.
94. I find driving on the left in England very strange. (acustomed)
I am …………………………………………………………………….. on the left in England.
95. She always has a good relationship with the children. (gets)
She always ……………………………………………………………………. the children.

SỞ GIÁO DỤC VÀ ĐÀO KỲ THI CHỌN HỌC SINH GIỎI


TẠO CẤP TỈNH LỚP 9 NĂM 2018

MÔN: TIẾNG ANH


(Đề thi gồm có 09 trang) Thời gian làm bài: 120 phút (không kể thời gian phát đề)
Ngày thi: ……………………………………
ĐỀ 2
SECTION I: LISTENING (3.0 points) (You will hear twice for each part)

PART 1. You will hear a man called Ian talking on the radio about difficult neighbours. For
each question, choose the correct answer.
1. Why couldn't Isabel sleep?
A. Her flat-mate was too noisy.
B. The phone kept ringing
C. The downstairs neighbor was shouting.
2. Isabel was angry because ________.
A. this had happened many times before.
B. the man was shouting at her
C. people were phoning her late at night.
3. What happened when Isabel approached the man?
A. He hit her. B. He reluctantly apologized C. He wasn't at all sorry
4. Why did Isabel move?
A. The men followed her home from work
B. Nobody would do anything about the man.
C. The renting agency asked her to.
5. According to research ________.
A. one in ten people argue with their neighbors.
B. one in ten people are disturbed by noise.
C. one in ten people are forced to move home.
6. According to Lisa Dorn ________.
A. modern living conditions cause problems.
B. people no longer know their neighbors.
C. people are more lonely than they used to be.
Part 2. You will hear a tour guide talking to a group of people. For each question, fill in the
missing information in the numbered space.
EXCURSION TO BRIGHTON
ITINERARY
COACH PICK UP TIME: 8 a m
PICK UP POINT: outside the (7) …………………………
ARRIVAL TIME IN BRIGHTON: (8) …………………………
DROP OFF POINT: Pool Valley Coach Station
guided walking tour
10.15 - 10.45: Tour of the famous Brighton Lanes Famous for (9) …………………………and
Boutiques
11 am - 12.30 pm: Coffee break. Refreshments are available inside the Palace Café.
or in the (10) …………………………café.
11.30 - 12.30: Guided tour of the palace.
12.45 - 2 pm: lunch at Donatello Restaurant.
Two-course lunch (11) £ …………………………
Three-course lunch (12) £ …………………………
Coffee /tea included
2pm - 3 p m: Brighton Museum and Art Gallery.
Guided tour optional- free of charge.
3pm - 5 pm: Free time on Brighton seafront.
Recommended sights: Brighton Pier (13) …………………………and artists’
studios.
5.10 - 6 p m: Grand Hotel for (14) …………………………
Depart from Pool Valley (15) ………………………….
SECTION II: LEXICO – GRAMMAR (7.0 points)
Part 1: Questions from 16 to 36. (3.0 points - 0.15/ each)
Choose the best answer to each question.
16. There are over 2,000 varieties of snakes, ________are harmless to humans.
A. mostly they B. most of them C. most of which D. which most
17. Smoke jumpers are ________descend into remote areas by parachute to fight forest fires.
A. firefighters B. when firefighters C. who, as firefighters D. firefighters who
18. It was the ragtime pianist Scott Joplin ________the "Maple Leaf Rag," perhaps the best
known of all ragtime tunes. Giaoa nde thitienganh.in fo

A. wrote B. the writer of C. who wrote D. writing


19. In geometry, a tangent is a straight line ________a curve at only one point.
A. it touches B. whose touching C. its touching D. that touches
20. The size and shape of a nail depends primarily on the function ________intended.
A. which it is B. for which it is C. which it is for D. for which is
21. A keystone species is a species of plant or animal ________absence has a major effect on
an ecologial system.
A. that its B. its C. whose D. with its
22. Seals appear clumsy on the land, ________are able to move short distances faster than most
people can run.
A. but they B. which they C. they D. which
23. Most folk songs are ballads ________have simple words and tell simple stories.
A. what B. although C. who D. that
24. ________added to a liquid, antifreeze lowers the freezing temperature of that liquid.
A. That B. As is C. It is D. When
25. ________ through a prism, a beam of white light breaks into all the colors of the rainbow.
A. When shines B. It shines C. It is shone D. When shone
26. Well- ___________child often behaves quite differently from one who did not get good
schooling.
A. education B. educated C. educate D. educating
27. When my grandfather was alive, he ________ morning exercises.
A. used to do B. use to do C. used to doing D. wasn't used to do
28. There is no __________ explanation for what happened.
A. scientifically B. scientist C. science D. scientific
29. –“I’ve passed my driving test.” –“_____________.”
A. It’s nice of you to say so. B. Do you?
C. Congratulations! D. That’s a good idea.
30. We ______ them before the reception yesterday
A. haven't met B. hadn't met C. didn't meet D. wouldn't meet
31. I could not _____ the lecture at all. It was too difficult for me.
A. make off B. take in C. get along D. hold on
32. Some sociologists believe that the _____ family is rapidly becoming a thing of the past.
A. basic B. closed C. immediate D. nuclear
33. Bill: “I'm having some friends over for lunch this saturday. Would you like to join us?'' Nancy:
“_________.”
A. Can I take a rain check? B. Come on. it's your turn
C. Thanks, but I mustn't D. As a matter of fact, I do
34. The criminal knows the ________ of successful robberies.
A. trash and treasure B. part and parcel C. ins and outs D. close all
35. To buy this type of product, you must pay half of the money ________, and pay the rest on the
day of delivery.
A. in advance B. in cheque C. in cash D. in charge
Part 2: Questions from 36 to 45. (2 points - 0.2/ each)
Use the correct form of the word in brackets to complete the following sentences.
36. He's a really …………………………person. RELY
37. She made several excellent …………………………in her essay on Charles Dickens. OBSERVE
38. Watching TV shows in English is definitely very …………………………to improve your USE
listening skills.
39. The Internet is probably one of the best …………………………ever. INVENT
40. The boat was 16 feet in …………………………. LONG
41. Both …………………………and fluency are important when speaking a foreign ACCURATE
language.
42. You shouldn't have said that! It was a totally …………………………remark. APPROPRIATE
43. Since the earliest times, civilisations have understood the …………………………of IMPORTANT
time.
44. Doing puzzles keeps our brains fit and …………………………. HEALTH
45. As well as gaining …………………………, by doing puzzles we give our brains a good SATISFY
workout.
Part 3: Questions from 46 to 50.(1 point - 0.2/ each)
Each of the following sentences has an error. Find the errors and correct them.
46. The notebook lists every opportunities for handicapped workers in the area.
47. In the end, she quit the job because it was too bored.
48. Every candidate under considering for a federal job must undergo a thorough medical
examination.
49. When I was a child, summers would be warmer and winters colder than now.
50. It took them at least two months learning how to pronounce these words.
Part 4: Questions from 51 to 55.(1 point - 0.2/ each)
Fill in each of the sentences with correct prepositions.
51. Our teacher is really intolerant …………………anyone who has a different point of view to her
own.
52. So …………………what grounds are you claiming compensation?
53. Yes, but …………………respect, you did miss the last meeting, didn't you?
54. Joey came …………………a very interesting book the other day.
55. Mary will not be able to come; she has come …………………with a flu.
SECTION III: READING (6.0 points)
Part 1. Questions 56 – 65 (2 points - 0.2/ each)
Read the passage below and choose the best answer to each question.
The first question we might ask is: What can you learn in college that will help you in being
an employee? The schools teach a (56) ________ many things of value to the future
accountant, doctor or electrician. Do they also teach anything of value to the future
employee? Yes, they teach the one thing that it is perhaps most valuable for the future
employee to know. But very few students bother (57) ________it. This basic is the skill ability
to organize and express ideas in writing and in speaking. This means that your success as an
employee will depend on your ability to communicate, with people and to (58) ________your
own thoughts and ideas to them so they will (59) ________understand what you are driving
at and be persuaded.
Of course, skill in expression is not enough (60) ________itself. You must have something
to say in the first place. The effectiveness of your job depends (61) ________your ability to
make other people understand your work as they do on the quality of the work itself.
Expressing one's thoughts is one skill that the school can (62) ________teach. The
foundations for skill in expression have to be (63) ________early: an interest in and an ear
(64) ________language; experience in organizing ideas and data, in brushing aside the
irrelevant, and above all the habit of verbal expression. If you do not lay these foundations
(65) ________your school years, you may never have an opportunity again.
56. A. large B. great C. far D. lots
57. A. learning B. to learn C. with learning D. learn
58. A. interpret B. give out C. transfer D. present
59. A. both B. not C. as well D. either
60. A. on B. for C. by D. in
61. A. on most B. most on C. much on D. on much
62. A. quite B. hardly C. truly D. really
63. A. liedB. laid C. lain D. lay
64. A. by B. in C. for D. of
65. A. during B. of C. for D. when
Part 2. Questions 66 – 75 (2.0 points - 0.2/ each)
Read the passage below and choose the best answer to each question
Although they are an inexpensive supplier of vitamins, minerals, and high-quality protein,
eggs also contain a high level of blood cholesterol, one of the major causes of heart diseases.
One egg yolk, in fact, contains a little more than two-thirds of the suggested daily cholesterol
limit. This knowledge has caused egg sales to plummet in recent years, which in turn has
brought about the development of several alternatives to eating regular eggs. One alternative
is to eat substitute eggs. These egg substitutes are not really eggs, but they look somewhat
like eggs when they are cooked. They have the advantage of having low cholesterol rates, and
they can be scrambled or used in baking. One disadvantage, however, is that they are not
good for frying, poaching, or boiling. A second alternative to regular eggs is a new type of egg,
sometimes called 'designer' eggs. These eggs are produced by hens that are fed low-fat diets
consisting of ingredients such as canola oil, flax, and rice bran. In spite of their diets, however,
these hens produce eggs that contain the same amount of cholesterol as regular eggs. Yet,
the producers of these eggs claim that eating their eggs will not raise the blood cholesterol in
humans.
Egg producers claim that their product has been portrayed unfairly. They cite scientific
studies to back up their claim. And, in fact, studies on the relationship between eggs and
human cholesterol levels have brought mixed results. It may be that it is not the type of egg
that is the main determinant of cholesterol but the person who is eating the eggs. Some
people may be more sensitive to cholesterol derived from food than other people. In fact,
there is evidence that certain dietary fats stimulate the body's production of blood
cholesterol. Consequently, while it still makes sense to limit one's intake of eggs, even
designer eggs, it seems that doing this without regulating dietary fat will probably not help
reduce the blood cholesterol level.
66. What is the main purpose of the passage?
A. To introduce the idea that dietary fat increases the blood cholesterol level.
B. To inform people about the relationship between eggs and cholesterol.
C. To persuade people that eggs are unhealthy and should not be eaten
D. To convince people to eat 'designer' eggs and egg substitutes.
67. According to the passage, which of the following is a cause of heart diseases?
A. minerals B. cholesterol C. canola oil D. vitamins
68. Which of the following could best replace the word 'somewhat'?
A. in fact B. a little C. indefinitely D. a lot
69. What has been the cause for changes in the sale of eggs?
A. increasing price B. decreased production
C. dietary changes in hens D. concerns about cholesterol
70. According to the passage, one yolk contains approximately what fraction of the suggested
daily limit for human consumption of cholesterol?
A. 3/4 B. 2/3 C. 1/2 D. 1/3
71. The word 'portrayed' could best be replaced by which of the following?
A. studied B. destroyed C. tested D. described
72. What is the meaning of 'back up'?
A. reverse B. advance C. block D. support
73. What is meant by the phrase 'mixed results'?
A. The results are blended. B. The results are a composite of things.
C. The results are inconclusive. D. The results are mingled together.
74. According to the passage, egg substitutes cannot be used to make any of following types
of eggs EXCEPT?
A. boiled B. poached C. scrambledD. fried
75. According to the author, which of the following may reduce blood cholesterol?
A. reducing egg intake but not fat intake B. increasing egg intake and fat intake
C. decreasing egg intake and fat intake D. increasing egg intake but not fat intake
Part 3: Questions 76 – 85 (2.0 points - 0.2/ each)
Read the text and fill in one word which best fits each gap.
Miss Darby was one of those people who never threw anything away. “You never know
when you might need it” was (76) …………………of her favorite sayings. She lived (77) …………………herself
in a large Victorian house across the road from us. Although I never went to her house, I knew
it was full (78) …………………antique furniture, Persian carpets and so on. In every room, there were
dozens of paintings so that her house was (79) …………………an art gallery. I remember my father
(80) …………………that she was a “Staffordshire Darby” but I had (81) …………………idea what he meant. I
(82) …………………out years later that the Darby family had made their money from coal-mining in
Staffordshire. We used to make up stories (83) …………………her. My sister Alice, (84) …………………was a
romantic girl, told us that Miss Darby once had a lover, but he walked out one day and she
(85) …………………saw him again!
Part 1:Questions from 86 to 90.(1 point- 0.2/ each)
Finish each of the following sentences in such a way that it means the same as the sentence
printed before it.
86. I am having a lot of trouble now because I lost my passport last week.
If I
87. She asked an assistant how much the Colombian coffee cost.
She asked, “
88. Although it looks fierce, the dog has never bitten anyone.
Fierce
89. "You should have waited for us," the team leader said to John.
The team leader criticized
90. As people use a lot of wood-pulp, many trees are cut down.
The more
Part 2:Questions from 91 to 95.(0.5 point - 0.25/ each)
Complete the second sentence so that it has a similar meaning to the first sentence, using
the word given. Do not change the word given.
91. Most people regard him as the best man for the job. (widely)
He is …………………………………………………………………………man for the job.
92. The suspect could not explain why he had sand on his boots. (account)
The suspect could …………………………………………………………………………on his boots.
93. The best solution was thought of by Sally. (came)
Sally …………………………………………………………………………solution.
94. You should consider the fact that he hasn’t spoken French for years. (allowances)
You should …………………………………………………………………………speaking French for years.
95. House prices have risen sharply this year. (increase)
There has …………………………………………………………………………this year.

……………………THE END ……………………...


- Thí sinh không được sử dụng tài liệu
- Giám thị không giải thích gì thêm

SỞ GIÁO DỤC VÀ ĐÀO TẠO KỲ THI CHỌN HỌC SINH GIỎI


CẤP TỈNH LỚP 9 NĂM 2018

(Đề thi gồm có 09 trang) MÔN: TIẾNG ANH


Thời gian làm bài: 120 phút (không kể thời gian phát đề)
Ngày thi: …………………………………
ĐỀ 3

SECTION I: LISTENING (3.0 points) (You will hear twice for each part)
Part 1. You will hear a man describing a series of books. For each Question, put a tick in the
correct box.
1. Why does he like 'The Planet Wars'?
A. It has a believable plot. B. The story is fascinating. C. The characters are deeply emotional.
2. What is his problem with 'ALong Way Horns'?
A. It was too long. B. The idea was bad. C. The writing was bad.
3. What does he say is original about 'Catch a Dream”
A. The happy ending.
B. The megical abilities of the characters.
C. The changing emotions of the characters.
4. The horror story is about
A. a war between humans and vampires.
B. Dracula is trying to use science to take over the world.
C. a battle between two groups of vampires.
5. What is not true about 'A World of Stories'?
A. It is a collection of different children's stories.
B. Forty children from around the world tell their life stories.
C. Stories are included fram different continents.
6. What does he think about 'The Real Shakespeare"
A. It does not contain accurate facts
B. It will be remembered as a classic
C. It will change people's minds about Shakespeare.
Part 2. You will hear someone talking on the radio about taxis and private hire vehicles in
London. For each question, fill in the missing information in the numbered space.
G iaoandethitie nganh.inf o

LONDON TAXIS AND PRIVATE HIRE VEHICLES


Taxi Services: Taxi; & private hire 24 hours a day, 365 days a year.
Pay in (7) ……………..………….. or with credit & debit cards.
Not all black
Stopped in the (8) ……………..………….. or at cab ranks.
Can be booked (9) ……………..…………...
Taxi Costs Depends on time of day, (10) ……………..………….. travelled and time taken.
Tariff 1: Monday - Friday (11) ……………..…………...
Tarifif 2: Monday - Friday 8 p.m. - 10 p.m., (12) ……………..…………..
Tariff 3 Every night 10 p.m. 6 am & on (13) ……………..…………..
Tests to become a Taxi Driver
Criminal record check
(14) ……………..………….. examination.
Knowledge of London's streets
Private Hire Vehicles
Limousine
Chauffer services, often known as (15) ……………..…………..
Journeys always booked in advance by visiting office or by phone
SECTION II: LEXICO – GRAMMAR (7.0 points)
Part 1: Questions from 16 to 36. (3.0 points - 0.15/ each)
Choose the best answer to each question.
16. ________the outer rings of a gyroscope are twisted or turned, the gyroscope itself
continues to spin in exactly the same position.
A. Somehow B. OtherwiseC. However D. No matter
17. Scientists cannot agree on _____________ related to other orders of insects.
A. that fleas are B. how fleas are C. how are fleas D. fleas that are
18. The authorities are determined to take tougher__________to reduce crime.
A. situations B. measures C. requirements D. interests
19. The old theater is in need of complete__________.
A. renew B. rebuild C. renovation D. review
20. Prior to the economic reform, the country had experienced many years of a __________
economy.
A. stagnant B. fluent C. fluently D. smoothly
21. Education is our first__________now. We are trying to do what we can to make it better.
A. prior B. prioritization C. priory D. priority
22. They have made a substantial change; that means, they have made a(n)
__________change.
A. economic B. considerable C. steady D. subtle
23. M.D. played much better than their opponents; they almost__________the match.
A. dominated B. eliminated C. activated D. terminated
24. The company was__________because of a shortage of orders.
A. solved B. resolved C. dissolved D. involved
25. It looks like they're going to succeed __________their present difficulties.
A. because of B. despite C. yet D. even though
26. In spite of______, we decided to go out.
A. tired B. feeling tired C. all of us felt tiredD. we felt very tired
27. “Do you mind if I borrow a chair?" “____ Do you need only one?”
A. Yes, I would. B. Yes, I do. C. Not at all. D. I'm sorry.
28. My parents saw a ________ play last night.
A. three - act B. three - acts C. three acts D. three act
29. Lucy was late for school this morning because the alarm didn’t ______ as usual
A. get off B. go off C. ring off D. take off
30. These books belong to you. I’m sure they are_____.
A. you’re B. your’s C. yours D. your
31. The other day I came________ a really beautiful old house in the back streets of London.
A. over B. down C. by D. across
32. He was looked _____ by others because of his poverty.
A. out for B. up to C. down upon D. into
33. He bought the cottage__________renovating it and then selling at a large profit.
A. with a view to B. with a target of C. his goal being D. aiming to
34. She _____ down completely on hearing of her friend's death.
A. broke B. turned C. put D. fell
35. We intend to _________with the old system as soon as we have developed a better one
A. do up B. do away C. do in D. do down
Part 2: Questions from 36 to 45. (2 points - 0.2/ each)
Use the correct form of the word in brackets to complete the following sentences.
36. The most ……………..………….. games have sold in the millions. SUCCESS
37. People have ……………..………….. that doing puzzles is good for you. COVER
38. It's easy to find a ……………..………….. to the problems posed. SOLVE
39. There has been an ……………..………….. in the power of their brains. IMPROVE
40. Some ……………..………….. argue that the brain gets better at a task the more it repeats SCIENCE
it.
41. The improvement in the ……………..………….. is something that happens naturally. PERFORM
42. It remains ……………..………….. whether puzzles are actually helping to boost CERTAIN
brainpower or not.
43. According to ……………..………….., most people sleep less than 8 hours every day. SEARCH
44. The advice passed down to our grandparents may contain some ……………..…………... TRUE
45. A good example is the ……………..………….. between being cold and catching a cold. RELATION
Part 3: Questions from 46 to 50.(1 point - 0.2/ each)
Each of the following sentences has an error. Find the errors and correct them.
46. When I was a small child, there would be a big banyan tree at the entrance to my village.
47. The lion has long been a symbol of strength, power, and cruel.
48. We took part in a six-hours journey to the eastern coast.
49. Different kinds of newspapers are arranging on the shelves.
50. There are a little activities that we can do after the meeting.
Part 4: Questions from 51 to 55.(1 point - 0.2/ each)
Fill in each of the sentences with correct prepositions.
51. I read a great book ……………..………….. John Grisham.
52. He's Italian? For some reason I was ……………..………….. the impression that he was Spanish.
53. We've put a lot of effort ……………..………….. this project.
54. After his father died, Nelson came ……………..………….. a fortune.
55. I was born in London but I was brought ……………..………….. in Madrid.
SECTION III: READING (6.0 points)
Part 1. Questions 56 – 65 (2 points - 0.2/ each)
Read the passage below and choose the best answer to each question.
ROBOTS
Ever since it was first possible to make a real robot, people have been hoping for the
invention of a machine that would do all the necessary jobs (56) ________ the house. If
boring and repetitive factory work could be (57) ________ by robots, why not boring and
repetitive household chores too?
For a long time the only people who really (58) ________ the problem their attention
were amateur inventors. And they came up against a major difficulty. That is, housework is
actually very complex. It has never been one job, it has always been many. A factory robot
(59) ________ one task endlessly (60) ________ it is reprogrammed to do something else. It
doesn’t run the whole factory. A housework robot, on the other hand, has to do several
different (61) ________ of cleaning and carrying jobs and also has to cope (62) ________ all
the diffenrent shapes and positions of rooms, furniture, ornaments, cats and dogs. (63)
________, there have been some developments recently. Sensors are available to (64)
________ the robot locate objects and avoid obstacles. We have the technology to produce
the hardware. All that is missing the software- the programs that will (65) ________ the
machine.
56. A. through B. over C. around D. for
57. A. managed B. succeeded C. made D. given
58. A. did B. took C. gave D. showed
59. A. carries over B. carries out C. carries off D. carries away
60. A. until B. while C. since D. when
61. A. systems B. methods C. ways D. types
62. A. from B. with C. by D. for
63. A. However B. Therefore C. Besides D. Moreover
64. A. enable B. help C. assist D. allow
65. A. practice B. perform C. order D. operate
Part 2. Questions 66 – 75 (2.0 points - 0.2/ each)
Read the passage below and choose the best answer to each question
Archimedes' Principle is a law of physics that states that when an object is totally or
partially immersed in a fluid, it experiences an upthrust equal to the weight of the fluid
displaced. The principle is most frequently applied to the behaviourof objects in water, and
helps to explain floating and sinking, and why objects seem lighter inwater. It also applies to
balloons. The key word in the principle is “upthrust”, which refers to the force acting upward
to reduce the apparent weight of the object whenit is under water. If, for example, a metal
block with a volume of 100 cm3 is dipped in water, it displaces an equal volumeof water,
which has a weight of approximately 1 N (3.5 oz). The block therefore seems to weigh about 1
N less.
An object will float if its average density is less than that of water. If it is totally
submerged, the weight of the water it displaces (and hence the upthrust on it) is greater than
its own weight, and it is forced upward and out of the water, until the weightof the water
displaced by the submerged part is exactly equal to the weight ofthe floating object. Thus a
block of wood with a density six tenths that of water will float with six tenths of its volume
under water, since at that point the weight of fluid displaced is the same as the block’s own
weight. If a dense material is made into a suitable shape, it will float because of Archimedes’
principle. A ship floats, whereas a block of iron of the same mass sinks.
It is also because of Archimedes’ principle thatships float lower in the water when they
are heavily loaded (more water must be displaced to give the necessary upthrust). In addition,
they cannot be so heavily loaded if they are to sail in fresh water as they can if they are to sail
in the sea, since fresh water is less dense than sea water, and so more water must be
displaced to give the necessary upthrust. This means the ship is lower in the water, which can
be dangerous in rough weather.
66. What happens when something is immersed in a fluid?
A. It will be pushed further down with a force, equal to the weight of the fluid displaced.
B. It receives an upward force, equal tothe weight of the fluid displaced.
C. It receives a downward force, equal tothe weight of the fluid displaced.
D. The fluid will expand the object and overflow to the floor.
67. The word “volume” in the passage refers to ______.
A. loudness B. quantity C. frequency D. length
68. The word “displaces” in the passage almost means “______”.
A. takes the place of B. takes place C. replaces with a new one D. puts in position
69. If an object’s average density is less than that of water, the object will ______.
A. sink B. float C. inflate D. drift
70. A block of wood with a density seven tenths that of water will ______.
A. go up and down then sink
B. float with a half of its volume under water
C. float with an equal volume of its volume under water
D. sink immediately when submerged
71. A ship floats, whereas a block of iron ofthe same mass sinks because the ship ______.
A. is made of wood B. is lighter C. has buoys D. has a special shape
72. The phrase “six tenths” in the passage means “______”.
A. 6 and 10 B. 10/6 C. 10 of 6 D. 6/10
73. The word “upthrust” in the passage refers to the ______.
A. upward push B. upper side of an object C. upturned force D. upside-down turn
74. Ships cannot be so heavily loaded if they want to sail in fresh water as they sail in the sea,
because ______.
A. fresh water is ‘lighter’ than sea water B. there’s too much salt in sea water
C. sea water is ‘saltier’ than fresh water D. fresh water is more polluted
75. Archimedes' Principle explains why ______.
A. all objects will float B. Archimedes became famous
C. objects seem lighter in water D. humans can swim
Part 3: Questions 76 – 85 (2.0 points - 0.2/ each)
Read the text and fill in one word which best fits each gap.
Watching Television and going for a walk are the most popular leisure (76) …………..…..…………..
in Britain. But although longer holidays and shorter working hours have given people more
free (77) ………………..………….., women generally have less free time (78) ……………..…..……….. men, because they
spend time (79) ………………..………….. domestic work, shopping and childcare. Surveys showed that
more men (80) ……………..………….. newspapers than women, and (81) ……………….………….. slightly higher
proportion of adult read Sunday newspapers than read (82) …………….…..………….. morning national
newspaper. More people are taking holidays abroad (83) ………………..………….. 1971 only 36 percent of
(84) …..…………..………….. in Britain had been abroad on holiday; but by 1983 this proportion had (85)
………………..………….. to 62 percent, nearly fifteen million people.
SECTION IV: WRITING (4.0 points)
Part 1: Questions from 86 to 90. (2 point- 0.4/ each)
Finish each of the following sentences in such a way that it means the same as the sentence
printed before it.
86. I left without saying goodbye as I didn’t want to disturb the meeting.
Rather
87. Everyone thinks I picked all the flowers in the garden.
I am thought
88. We decided to move to the countryside.
We made
89. They were never aware at any moment that something was wrong.
At no time
90. The heavy rain prevented everybody from going out.
Had
Part 2: Questions from 91 to 92.(2 pointS - 0.4/ each)
Complete the second sentence so that it has a similar meaning to the first sentence, using
the word given. Do not change the word given.
91. It was easy for us to get tickets for the concert. no
We ……………..……………………..…………..……………..………….. tickets for the concert.
92. The two theories appear to be completely different. common
The two theories ……………..……………………..…………..……………..…………...
93. I can’t describe people as well as you can. better
You’re ……………..……………………..…………..……………..………….. I am.
94. I don’t know how you can stand getting up so early to go to the pool. put
I don’t know how you ……………..……………………..…………..……………..………….. up so early to go to the pool.
95. “It was your fault to break my vase yesterday” said Jane to her brother. Accused
Jane accused ……………..……………………..…………..……………..………….. her vase the previous day.

SỞ GIÁO DỤC VÀ ĐÀO TẠO KỲ THI CHỌN HỌC SINH GIỎI


CẤP TỈNH LỚP 9 NĂM 2018

(Đề thi gồm có 09 trang) MÔN: TIẾNG ANH


Thời gian làm bài: 120 phút (không kể thời gian phát đề)
Ngày thi: ……………………………

ĐỀ 4

SECTION I: LISTENING (3.0 points) (You will hear twice for each part)
Part 1. You will hear a man, David, being interviewed about his life as a professional
footballer, for each question, put a tick in the correct box.
1. How long has David been a professional football play?
A. One match B. Two years C. four years
2. What is in a normal day for David?
A. Fitness training and tactics.
B. fitness training and a full match.
C. Fitness, training tactics and a full match.
3. What do the team not watch videos about?
A. The opposition. B. Warming up. C. Their own performance.
4. What does David say about the diet of a footballer?
A. it is often unpleasant and bad.
B. It has lots of rice, meat and pasta.
C. Footballers have to be careful about what they eat.
5. What is true about David's free time?
A. He spends most of his free time with his friends
B. He has very little free time, except in the summer.
C. He usually does not manage to see his family.
6. What does David say about his future ambitions?
A. He firstly wants to secure a regular place in the team.
B. He wants to play for a European team in the next two years.
C. He never thinks about playing in the World Cup.
Part 2. You will hear a woman talking on the radio about fours of historic houses in the
south of England, for each question, fill in the missing information in the numbered space.
HISTORIC TOURS
South Elmharn House. Giaoan de thitienga nh.info
Built: (7) …………………..……… century by the bishops of Norwich
Improved: 16th century by a group of rich (8) …………………..………
Features: many old, valuable, wall paintings.
Remains of a small Norman (9) …………………..………
Tours: Including a walk 2 p.m £ 12. tea / coffee, meal bookable.
Haughley Hall
Built: 14th century outside ruined castle
Improved: 18th century.
Features: Once owned by (10) …………………..……… II, currently owned by a Lord.
Secret (11) …………………..……… in the walls
Two tunnels now closed
Tours 11.30 or 2 p.m £ with traditional (12) …………………..………, £ 12 with tea. Group
welcome.
Bedfield House
Built: (13) …………………..………century by the church.
Improved: Mid 15th century.
Features Signs that protect against witchcraft are on (14) …………………..………and surfaces.
Gardens are joins by (15) …………………..………
Tours: 10.30 a.m or 2.30 p.m, £13.50 with tea/coffee and cakes. Groups welcome.
SECTION II: LEXICO – GRAMMAR (7.0 points)
Part 1: Questions from 16 to 36. (3.0 points - 0.15/ each)
Choose the best answer to each question.
16. I'm expecting to have this book__________ soon this year.
A. published B. publishing C. to publish D. publish
17. ________ your advice, I was able to avoid lots of trouble on my trip abroad.
A. Thanks to B. Despite C. Even though D. Instead of
18. We have a party tonight and Daisy is worried about ________.
A. what to wear B. which wearing C. that she wears D. these wearing
19. As a rule, new comers ________ a party at the end of the first month of their stay.
A. was held B. hold C. have held D. will hold
20. The most interesting films ________ for the festival will be shown next week.
A. are chosen B. having chosen C. chosen D. being chosen
21. However ________ about the results, the children still talk merrily.
A. disappointed are they B. disappointing they are
C. disappointment D. disappointed they are
22. A child ________ to talk does not learn by being corrected all the time.
A. learned B. learns C. learning D. to learn
23. Some businesses use famous cartoon ________ to advertise their products.
A. person B. people C. charactersD. samples
24. The first thing you should do when you lose your bank card is to call your bank and ask
them to close your ________.
A. account B. house C. loan D. debt
25. “Oh no! I can’t find my credit card!” “________.”
A. Thank you for letting me know
B. Don’t worry. They will be back very soon
C. It is a nice surprise. You should be glad about it
D. Chill out. Try to remember when you last used it
26. – “What would you like to drink? “ – “ __________.”
A. Yes, please B. Milk, please C. No, thanks D. OK
27. The number of cars on the roads ______ increasing, so we need to build more roads.
A. is B. wasC. were D. are
28. ______ the class size is our school’s immediate aim.
A. Reduces B. Reduce C. Reducing D. Reduced
29. He ________ being given a receipt for the bill he had paid.
A. asked to B. demanded C. insisted on D. required
30. Some________ actions have been taken to help protect wildlife at this national park.
A. survival B. disposal C. postal D. remedial
31. The demand for energy is ________more rapidly than ever.
A. exported B. decreased C. raising D. increasing
32. His performance was________; the audience was delighted.
A. unmarked B. faultless C. worthless D. imperfect
33. But why did the police suspect you? It just does not make________ to me.
A. reason B. right C. sense D. truth
34. In the end I ________the form in disgust, and threw it away.
A. filled in B. worked out C. tore up D. put off
35. His company, ________him with a car and samples of their products.
A. gives B. supplies C. replies D. places
Part 2: Questions from 36 to 45. (2 points - 0.2/ each)
Use the correct form of the word in brackets to complete the following sentences.
36. These high-heeled shoes are …………………..……… for such rough terrain. SUIT
37. Colds are caused by viruses, so in the …………………..……… of a virus, you can't catch ABSENT
a cold.
38. You're too young. This film is …………………..……… for children your age. APPROPRIATE
39. …………………..……… now think that we may have viruses in our bodies already. SCIENCE
40. In cold weather, for example, blood vessels in the nose get smaller to stop FORTUNATE
heat escaping. …………………..………, this also allows the cold virus to attack the nose or
throat more easily.
41. Victoria Falls in Africa is one of the most …………………..……… sights in the world. SPECTACLE
42. It's also an …………………..……… place for water sports. CREDIBLE
43. We need you to provide an accurate …………………..……… of the situation. DESCRIBE
44. On arrival, it's the noise that makes the greatest …………………..………. IMPRESS
45. The …………………..……… landscape is also well worth a visit. SURROUND
Part 3: Questions from 46 to 50.(1 point - 0.2/ each)
Each of the following sentences has an error. Find the errors and correct them.
46. Hung really wishes that he can take part in the game.
47. They’re going to get their house decorate for the May Day.
48. I used to sit next to a man who name is Almed.
49. I’d rather typing this letter than write it because it’s faster.
50. He drives too fast that no one likes to ride with him.
Part 4: Questions from 51 to 55.(1 point - 0.2/ each)
Fill in each of the sentences with correct prepositions.
51. It's about time you told him the truth!
52. The soccer player was ejected because he had done something that was against the rules.
53. Being a nurse is hard work, especially if you're on call all the time.
54. You’ll have to work very hard if you want to catch up with the rest of your classmates.
55. I came across my lost earring while I was sweeping in the floor.
SECTION III: READING (6.0 points)
Part 1. Questions 56 – 65 (2 points - 0.2/ each)
Read the passage below and choose the best answer to each question.
The first question we might ask is: What can you learn in college that will help you in being
an employee? The schools teach a (56) ________ many things of value to the future
accountant, doctor or electrician. Do they also teach anything of value to the future
employee? Yes, they teach the one thing that it is perhaps most valuable for the future
employee to know. But very few students bother (57) ________it. This basic is the skill ability
to organize and express ideas in writing and in speaking. This means that your success as an
employee will depend on your ability to communicate, with people and to (58) ________ your
own thoughts and ideas to them so they will (59) ________ understand what you are driving
at and be persuaded.
Of course, skill in expression is not enough (60) ________ itself. You must have something
to say in the first place. The effectiveness of your job depends (61) ________ your ability to
make other people understand your work as they do on the quality of the work itself.
Expressing one's thoughts is one skill that the school can (62) ________ teach. The
foundations for skill in expression have to be (63) ________ early: an interest in and an ear
(64) ________ language; experience in organizing ideas and data, in brushing aside the
irrelevant, and above all the habit of verbal expression. If you do not lay these foundations
(65) ________ your school years, you may never have an opportunity again.
56. A. large B. great C. far D. lots
57. A. learning B. to learn C. with learning D. learn
58. A. interpret B. give out C. transfer D. present
59. A. both B. not C. as well D. either
60. A. on B. for C. by D. in
61. A. on most B. most on C. much on D. on much
62. A. quite B. hardly C. truly D. really
63. A. liedB. laid C. lain D. lay
64. A. by B. in C. for D. of
65. A. during B. of C. for D. when
Part 2. Questions 66 – 75 (2.0 points - 0.2/ each)
Read the passage below and choose the best answer to each question
In 1826, a Frenchman named Niépce needed pictures for his business. He was not a
good artist, so he invented a very simple camera. He put it in a window of his house and took
a picture of his yard. That was the first photograph. The next important date in the history of
photography was 1837. That year, Daguerre, another Frenchman, took a picture of his studio.
He used a new kind of camera and a different process. In his
pictures, you could see everything clearly, even the smallest details. This kind of photograph
was called a daguerreotype.
Soon, other people began to use Daguerre's process. Travelers brought back
daguerreotypes from all around the world. People photographed famous buildings, cities, and
mountains.
In about 1840, the process was improved. Then photographers could take pictures of
people and moving things. The process was not simple and photographers had to carry lots of
film and processing equipment. However, this did not stop photographers, especially in the
United States. After 1840, daguerreotype artists were popular in most cities.
Matthew Brady was one well-known American photographer. He took many portraits
of famous people. The portraits were unusual because they were lifelike and full of
personality. Brady was also the first person to take pictures of a war. His 1862 Civil War
pictures showed dead soldiers and ruined cities. They made the war seem more real and
more terrible.
In the 1880s, new inventions began to change photography. Photographers could buy
film ready-made in rolls, instead of having to make the film themselves. Also, they did not
have to process the film immediately. They could bring it back to their studios and develop it
later. They did not have to carry lots of equipment. And finally, the invention of the small
handheld camera made photography less expensive.
With a small camera, anyone could be a photographer. People began to use cameras
just for fun. They took pictures of their families, friends, and favorite places. They called these
pictures "snapshots".
Documentary photographs became popular in newspapers in the 1890s. Soon
magazines and books also used them. These pictures showed true events and people. They
were much more real than drawings.
Some people began to think of photography as a form of art. They thought that
photography could do more than show the real world. It could also show ideas and feelings,
like other art forms.
66. The first photograph was taken with ______.
A. a small handheld camera B. a very simple camera
C. a daguerreotype D. new types of film
67. Daguerre took a picture of his studio with ______.
A. a new kind of camera B. a very simple camera
C. special equipment D. an electronic camera
68. The word “this” in the passage refers to the ______.
A. carrying of lots of film and processing equipment
B. stopping of photographers from taking photos
C. fact that daguerreotype artists were popular in most cities
D. taking of pictures of people and moving things
69. The word “ruined” in the passage is closest in meaning to “______”.
A. poorly-painted B. heavily-polluted C. terribly spoiled D. badly damaged
70. The word “lifelike” in the passage is closest in meaning to “______”.
A. moving B. realistic C. touching D. manlike
71. The latest invention mentioned in the passage is the invention of ______.
A. handheld cameras B. processing equipment
C. daguerreotypes D. rolls of film
72. The word “handheld” in the passage is closest in meaning to “______”.
A. handling manually B. held by hand C. controlling hands D. operated by hand
73. Matthew Brady was well-known for ______.
A. inventing daguerreotypes B. the small handheld camera
C. taking pictures of French cities D. portraits and war photographs
74. As mentioned in the passage, photography can ______.
A. print old pictures B. convey ideas and feelings
C. show the underworld D. replace drawings
75. Which of the following could best serve as the title of the passage?
A. Different Steps in Film Processing B. Story of Photography
C. Photography and Painting D. Story of Famous Photographers
Part 3: Questions 76 – 85 (2.0 points - 0.2/ each)
Read the text and fill in one word which best fits each gap.
SAFE CAMPING
Camping in the country is usually great fun, but sometimes things can go wrong.
Accidents can happen, so it is essential to think about safety both before you go ( 76) …………………..
……… while you are there. This will prevent your fun camping trip turning (77) …………………..………
something less pleasant.
Firstly, you need to plan ahead. Check out the weather (78) …………………..……… a few days in
advance and watch out for any reports of fires in the area you are thinking of going to. Bring
an emergency kit in (79) …………………..……… you or anyone with you has an accident or illness while
you are there.
Choose your camp (80) …………………..……… carefully, avoiding any places where there is risk of
flooding before you put up your tent, make (81) …………………..……… there are no sharp objects on the
ground, or ants’ or wasps’ nests nearby.
In order to keep insects out of the tent, close it whenever you go in or out. If you need
a camp (82) …………………..……… for cooking, be careful not to build it anywhere near your tent, and
before you go to bed, remember to put it (83) …………………..……… completely, preferably with lots of
water.
After meals, pick up any bits of food that may (84) …………………..……… left on the ground, as
these can attract insects – or larger creatures. It also makes senses, for the same reason, to
keep unused food in closed containers away from the camp. You don’t want a hungry bear or
another (85) …………………..……… suddenly appearing in your tent!
SECTION IV: WRITING (4.0 points)
Part 1: Questions from 86 to 90. (2 point- 0.4/ each)
Finish each of the following sentences in such a way that it means the same as the sentence
printed before it.
86. The result of the match was never in doubt.
At no
87. I didn’t know you were coming, so I didn’t wait for you.
If I
88. When I picked up my pen I found that the nib hadbroken
On
89. Sally finally managed to get a job.
Sally finally succeeded
90. Fiona was so disappointed that she could not keep on working.
Such
Part 2: Questions from 91 to 92.(2 pointS - 0.4/ each)
Complete the second sentence so that it has a similar meaning to the first sentence, using
the word given. Do not change the word given.
91. I haven’t decided to continue my study in a foreign country. made
I haven’t …………………..………………………………………..……… my study in a foreign country.
92. I can't understand Doris and what she has done. sense
I can't …………………..………………………………………..……… Doris and what she has done.
93. I think you should complain about that horrible meal. WERE
If I …………………..………………………………………..……… a complaint about that horrible meal.
94. I have a good relationship with my neighbors. ON
My neighbors …………………..………………………………………..……… together
95. I don’t usually remember people’s surnames. MEMORY
I don’t …………………..………………………………………..……… people’s surnames.

SỞ GIÁO DỤC VÀ ĐÀO TẠO KỲ THI CHỌN HỌC SINH GIỎI


CẤP TỈNH LỚP 9 NĂM 2018
MÔN: TIẾNG ANH
(Đề thi gồm có 08 trang) Thời gian làm bài: 120 phút (không kể thời gian phát đề)

ĐỀ 5

SECTION I: LISTENING (3.0 points) (You will hear twice for each part)

SECTION I. LISTENING. A - LISTENING COMPREHENSION (2pts)


PART 1. (0,5pt) You will hear five short conversations.
You will hear each conversation twice.
There is one question for each conversation.
For questions 1-5, put a tick under the right answer.
1. What is John going to do tonight?

2. Which is Ben’s family?

3. Which bag does the woman buy?

4. How much did the woman pay foe the apples?

5. What time does the film start?

PART 2(0. 5pt). Listen and write.

Oldest person: grandmother


1. Name: …………………………
2. Age: …………………………
3. Lives with: …………………………
4. Job: …………………………
5. Favourite hobby: …………………………
PART 3(1,0pt):Listen to Amanda talking to a friend about a birthday party. Tick A,B or C.
You will hear the conversation twice.
1. Which ice cream will they have at the party?
A. coffee B. Lemon C. apple
2. Which is broken?
A. the CD player B. the cassette recorder C. the guitar
3. Whose birthday is it?
A. Emma’s B. Joan’s C. Amanda’s sister’s
4. What present has Amanda bought?
A. a camera B. a video C. a football
5. What time should people arrive at the party?
A. 8 p. m B. 8. 30 p. m C. 9. 30 p. m
SECTION II: LEXICO – GRAMMAR (7.0 points)
Part 1: Questions from 16 to 36. (3.0 points - 0.15/ each)
Choose the best answer to each question.
16. I haven’t ________ decided where to go on holiday this year.
A. still B. yet C. already D. then
17. How do you speak this fraction: 3/5?
A. Three over fifth B. Three-fifth C. Three-fifths D. Three-fives
18. The number of cars on the roads ________ increasing, so we need to build more roads.
A. is B. wasC. were D. are
19. There is no ________ explanation for what happened.
A. scientifically B. scientist C. science D. scientific
20. I can ________ what he’s doing; it’s so dark down there.
A. see through B. make out C. look into D. show up
21. Could you be more specific about what is ________ in this particular job?
A. enclosed B. concentrated C. presented D. involved
22. He retired early ________ ill – health.
A. on behalf of B. on account of C. believe D. imagine
23. The person who writes symphonies or concertos is a ________.
A. composer B. conductor C. pianist D. playwright
24. There is no reason to______ his honesty; he is absolutely sincere.
A. search B. doubt C. inquire D. ask
25. His personal problems seem to have been ______him from his work lately.
A. disrupting B. disturbing C. distractingD. dispersing
26. The job of student lodgings officer________ many visits to landladies.
A. concerns B. offers C. asks D. involves
27. It is________ knowledge in the village that Mr. and Mrs. Thorne quarrel violently several
times a week.
A. common B. complete C. normal D. usual
28. “Which one do you want?” “I’m not sure I can ________a choice. I like all of them.”
A. do B. choose C. make D. take
29. The volcano had been _________ for hundreds of years before the eruption last month.
A. dying B. fixed C. asleep D. dormant
30. You have to ________the person with the ball until you catch them.
A. chase B. rush C. jump D. drop
31. The fans climbed over the fence to ________paying.
A. avoid B. prevent C. abandon D. refuse
32. I had no ________that the divorce rate was so high in this country.
A. knowledge B. idea C. doubt D. understanding
33. A railway bridge is already ________over the river.
A. been erected C. in construction B. erecting D. being built
34. It was very difficult for the inspector to________ what recommendations he should make.
A. realize B. settle C. solve D. decide
35. This is not the right________ to ask for my help; I am far too busy even to listen!
A. moment B. situation C. opportunity D. circumstance
Part 2: Questions from 36 to 45. (2 points - 0.2/ each)
Use the correct form of the word in brackets to complete the following sentences.
36. There's a post office a bit …………………………. down the road. FAR
37. Six months after the accident, he still has …………………………. walking. DIFFICULT
38. I can't play tennis that well because I'm a ………………………….. BEGIN
39. Don't go there. It's ………………………….. DANGER
40. Read this. It'll be very …………………………. for your trip to Australia. USE
41. I didn't find him that …………………………., but my friend did. ATTRACT
42. The school has a …………………………. for being very up-to-date with technology. REPUTE
43. These are …………………………. clothes and that's why they are very expensive. DESIGN
44. I left the party because it was …………………………. noisy in there. EXTREME
45. I had no …………………………. but to tell him. CHOOSE
Part 3: Questions from 46 to 50.(1 point - 0.2/ each)
Each of the following sentences has an error. Find the errors and correct them.
46. The novel writing by Charles Dickens attracts a lot of children.
47. Thank you very much for your letter, who came yesterday.
48. He experimented with ways of transmitting speech over a long distant with his assistant.
49. Each of the students in the accounting class has to type their own research.
50. Janet is finally used to cook on an electric stove after having a gas one for so long.
Part 4: Questions from 51 to 55.(1 point - 0.2/ each)
Fill in each of the sentences with correct prepositions.
51. In most countries, children start the primary school ………………. the age of six.
52. Since Jennifer was a year younger than her friends, she had a hard time keeping ………………. with
them.
53. There are different sets of language learning sets available ………………. all age groups.
54. I'll try to figure ………………. what's wrong with the computer tomorrow.
55. It took 10 firefighters to put ………………. the fire at the school.
SECTION III: READING (6.0 points)
Part 1. Questions 56 – 65 (2 points - 0.2/ each)
Read the passage below and choose the best answer to each question.
TRAFFIC IN OUR CITIES
The volume of traffic in many cities in the world today continues to expand. This causes
many problems, including serious air pollution, lengthy delays, and the greater risk (56)
_________ accidents. Clearly, something must be done, but it is difficult to (57) _________
people to change their habits and leave their cars at home.
One possible (58) _________ is to make it more expensive for people to use their cars by
(59) _________charges for parking and bringing in tougher fines for anyone who (60)
_________ the law. In addition, drivers could be required to pay for using particular routes at
different times of the day. This system, (61) _________ as “road pricing”, is already being
introduced in a number of cities, using a special electronic card (62) ______ to windscreen of
the car.
Another way of dealing with the problem is to provide cheap parking on the (63) _______
of the city, and strictly control the number of vehicles allowed in to the centre. Drivers and
their passengers then use a special bus service for the (64) _______ stage of their journey.
Of course, the most important thing is to provide good public transport. However, to get
people to give up the comfort of their cars, public transport must be felt to be reliable,
convenient and comfortable, with fares (65) __________ at an acceptable level.
56. A. of B. for C. about D. by
57. A. make B. arrange C. suggest D. persuade
58. A. approach B. manner C. custom D. style
59. A. enlarging B. increasing C. growing D. developing
60. A. crosses B. refuses C. breaks D. cracks
61. A. named B. seen C. called D. known
62. A. fixed B. joined C. built D. placed
63. A. outskirts B. border C. outside D. limit
64. A. lateB. endC. complete D. final
65. A. taken B. kept C. given D. stood
Part 2. Questions 66 – 75 (2.0 points - 0.2/ each)
Read the passage below and choose the best answer to each question
Although they are an inexpensive supplier of vitamins, minerals, and high-quality protein,
eggs also contain a high level of blood cholesterol, one of the major causes of heart diseases.
One egg yolk, in fact, contains a little more than two-thirds of the suggested daily cholesterol
limit. This knowledge has caused egg sales to plummet in recent years, which in turn has
brought about the development of several alternatives to eating regular eggs. One alternative
is to eat substitute eggs. These egg substitutes are not really eggs, but they look somewhat
like eggs when they are cooked. They have the advantage of having low cholesterol rates, and
they can be scrambled or used in baking. One disadvantage, however, is that they are not
good for frying, poaching, or boiling. A second alternative to regular eggs is a new type of egg,
sometimes called 'designer' eggs. These eggs are produced by hens that are fed low-fat diets
consisting of ingredients such as canola oil, flax, and rice bran. In spite of their diets, however,
these hens produce eggs that contain the same amount of cholesterol as regular eggs. Yet,
the producers of these eggs claim that eating their eggs will not raise the blood cholesterol in
humans.
Egg producers claim that their product has been portrayed unfairly. They cite scientific
studies to back up their claim. And, in fact, studies on the relationship between eggs and
human cholesterol levels have brought mixed results. It may be that it is not the type of egg
that is the main determinant of cholesterol but the person who is eating the eggs. Some
people may be more sensitive to cholesterol derived from food than other people. In fact,
there is evidence that certain dietary fats stimulate the body's production of blood
cholesterol. Consequently, while it still makes sense to limit one's intake of eggs, even
designer eggs, it seems that doing this without regulating dietary fat will probably not help
reduce the blood cholesterol level.
66. What is the main purpose of the passage?
A. To introduce the idea that dietary fat increases the blood cholesterol level.
B. To inform people about the relationship between eggs and cholesterol.
C. To persuade people that eggs are unhealthy and should not be eaten
D. To convince people to eat 'designer' eggs and egg substitutes.
67. According to the passage, which of the following is a cause of heart diseases?
A. minerals B. cholesterol C. canola oil D. vitamins
68. Which of the following could best replace the word 'somewhat'?
A. in fact B. a little C. indefinitely D. a lot
69. What has been the cause for changes in the sale of eggs?
A. increasing price B. decreased production
C. dietary changes in hens D. concerns about cholesterol
70. According to the passage, one yolk contains approximately what fraction of the suggested
daily limit for human consumption of cholesterol?
A. 3/4 B. 2/3 C. 1/2 D. 1/3
71. The word 'portrayed' could best be replaced by which of the following?
A. studied B. destroyed C. tested D. described
72. What is the meaning of 'back up'?
A. reverse B. advance C. block D. support
73. What is meant by the phrase 'mixed results'?
A. The results are blended. B. The results are a composite of things.
C. The results are inconclusive. D. The results are mingled together.
74. According to the passage, egg substitutes cannot be used to make any of following types
of eggs EXCEPT?
A. boiled B. poached C. scrambledD. fried
75. According to the author, which of the following may reduce blood cholesterol?
A. reducing egg intake but not fat intake B. increasing egg intake and fat intake
C. decreasing egg intake and fat intake D. increasing egg intake but not fat intake
Part 3: Questions 76 – 85 (2.0 points - 0.2/ each)
Read the text and fill in one word which best fits each gap.
All men should (76) ……………….………….. We study to (77) ……………….…………. our knowledge and
develop our intelligence. Education (78) ……………….…………. an important role in our life. First of all, we
have to learn (79) ……………….…………. to observe accurately, to think truthfully, to speak correctly and
to write clearly. Education gives us (80) ……………….…………. of things around us and it preserves the
national noble traditions and customs from generation to generation. Education makes a
person perfect. An (81) ……………….…………. man is both talented and virtuous. In every country, the
government consider education (82) ……………….…………. the most important policy. Some (83) ……………….
…………. us sometimes think that we have finished our education when we left school or (84)
……………….…………. from a university. Actually, real education should (85) ……………….…………. finish.
SECTION IV: WRITING (4.0 points)
Part 1: Questions from 86 to 90.(2 point- 0.4/ each)
Finish each of the following sentences in such a way that it means the same as the sentence
printed before it.
86. The phone stopped ringing the moment I got down stairs.
No sooner
87. He is determined to carry on working when he is 65.
- He has no
88. He was very sorry that he didn’t see Audrey on her trip to London.
- He greatly
89. She agreed to go out to dinner with him because she assumed he was not married.
- Had she
90. Everyone was surprised that the singer had very little money when he died.
- The singer had
Part 2: Questions from 91 to 92.(2 points - 0.4/ each)
Complete the second sentence so that it has a similar meaning to the first sentence, using
the word given. Do not change the word given.
91. Clara said that she had not seen the missing letter. having
- Clara ……………….………….……………….…………. the missing letter.
92. It's important to consider everyone's opinion before a final decision is made. account
- Everyone's opinion must ……………….………….……………….…………. before a final decision is made.
93. My passport needs renewing because I´m going abroad this summer. get
- I need ……………….………….……………….…………. because I´m going abroad this summer.
94. My boss doesn't mind what time I start or finish work. difference
- It ……………….………….…………………………….…………. what time I start or finish work.
95. Many people believe that Edison has the ability to become world judo champion. capable
- Many people believe that Edison ……………….………….……………….……the world championship in judo.

……………………THE END ……………………...


- Thí sinh không được sử dụng tài liệu
- Giám thị không giải thích gì thêm
SỞ GIÁO DỤC VÀ ĐÀO TẠO KỲ THI CHỌN HỌC SINH GIỎI
CẤP TỈNH LỚP 9 NĂM 2018
MÔN: TIẾNG ANH
(Đề thi gồm có 08 trang) Thời gian làm bài: 120 phút (không kể thời gian phát đề)
Ngày thi: 02/11/2017
ĐỀ 6

SECTION I: LISTENING (3.0 points) (You will hear twice for each part)
Part 1. You will hear a radio interview with the manager of a summer activity course. For
each question, choose the correct answer.
1. This year, the course will run for ________.
A. six weeks. B. seven weeks. C. eight weeks.
2. The problem last year was that________.
A. few people wanted to attend.
B. there were too few workers.
C. there was nothing to do on rainy days.
3. This, year, for the first time, children will________.
A. do creative activities. B. do new outdoor sports. C. organise events.
4. Molly doesn’t think children will come for six weeks because________.
A. it’s too expensive. Giaoa ndethitien ganh.info

B. they will do the same activities again and again.


C. their parents will want to spend time with them.
5. The course isn’t open to teenagers because________.
A. Molly thinks they aren’t interested in the activities which are available.
B. Molly thinks they should spend time with young people of a similar age.
C. Molly’s staff think that teenagers are difficult to please.
6. It’s important that parents of children attending the course________.
A. pay the full amount immediately.
B. choose the activities the child wants to do when they apply.
C. inform Molly about any food the child cannot eat.
Part 2. You will hear a radio announcer giving details about a local Motor Show. For each
question, fill in the missing information in the spaces. Write no more than three words
and/or a number.
Eastbrook Motor Show
Date: (7) ……………………………
Location: (8) …………………………… Show Ground
Displays: (9) …………………………… and buses
Cars of the (10) …………………………….
Famous People: Jack Tyler from the Television Show (11) ……………………………
Top Racing Driver Michael Boreman
For Ladies: Stalls selling (12) ……………………………, jewellery and clothes
Competition - Guess the number of (13) …………………………… in the car
£: (14) ……………………………pounds for adults and six pounds for children.
Price of Family
£: (14) ……………………………for two adults and up to three children for thirty
Ticket:
pounds
SECTION II: LEXICO – GRAMMAR (7.0 points)
Part 1: Questions from 16 to 36. (3.0 points - 0.15/ each)
Choose the best answer to each question.
16. He confessed _____ being somewhat nervous about having to speak in front of a large
crowd.
A. about B. to C. for D. with
17. In a money-oriented society, the average individual cares little about solving _____
problem.
A. any other B. any other’s C. anyone else’s D. anyone’s else
18. Would you please leave us details of your address _______ forwarding any of your mail to
come?
A. for the purpose of B. as a consequence of C. for the sake of D. by means of
19. The woman they finally choose was much older than the other_______ for the job.
A. informant B. participants C. applicants D. consultants
20. He can’t find money for life necessities, ______ such luxuries as wine and tobacco.
A. leave alone B. buy alone C. mention alone D. let alone
21. Cigarette smoking has been ______ breast as well as lung cancer.
A. concerned to B. originated from C. caused by D. associated with
22. The general is always ______ about his past campaigns.
A. boasting B. praising C. complimenting D. congratulating
23. ________ of the Chairman, the Executive Director will be responsible for chairing the
meeting.
A. For the absence B. On the absence C. In the absence D. To the absence
24. "I'm ________ to listen to your pathetic excuses," she said.
A. sick and tired B. in no mood C. having enough D. sick to death
25. Unfortunately our local cinema in on the ________ of closing down.
A. threat B. danger C. edge D. verge
26. I was early for the meeting so I went to a bookshop to ______ time.
A. kill B. waste C. spend D. make
27. This ticket _____ one person to the museum.
A. allows B. enters C. permits D. admits
28. You must _____ allowances for him; he is not very healthy.
A. get B. make C. do D. have
29. As the drug took _____, the boy became quieter.
A. force B. influence C. action D. effect
30. The teacher asked a difficult question, but finally Ted _________ a good answer.
A. put up with B. keep pace with C. made way for D. came up with
31. His brother refuses to even listen to anyone else's point of view. He is very ________.
A. kind - hearted B. narrow- minded C. open - minded D. absent - minded
32. The players’ protests_____ no difference to the referee’s decision at all
A. did B. made C. caused D. created
33. “I’m tired of the same old routine.” _ “I know how you feel. I get tired of doing the same
things day in_____.”
A. and day off B. and day on C. and day ofD. and day out
34. The ________of plastic bags gathered was nearly the same in both years.
A. number B. lot C. percent D. amount
35. After the accident, the injured cyclist was in great________.
A. agony B. suffering C. hurt D. pain
Part 2: Questions from 36 to 45. (2 points - 0.2/ each)
Use the correct form of the word in brackets to complete the following sentences.
36. This cloth is made from ……………………………fibers. NATURE
37. If you want to take care of the environment, it's important to ………………………. CYCLE
38. In the next few years, ………………………will hopefully have found a cure for malaria. SCIENCE
39. According to the ……………………………, the epidemic started in Kuala Lumpur. SEARCH
40. ……………………………from family and friends can lead to feelings of anxiety. ISOLATE
41. I was ……………………………to understand what the problem was. ABLE
42. There will be a ……………………………of solo games and competitive games available. MIX
43. Participants will later be asked for ……………………………on the event. FEED
44. It was an ……………………………reunion and we were all really touched when we EMOTION
remembered how we had met.
45. I am writing to ask for further ……………………………on your Open Day. INFORM
Part 3: Questions from 46 to 50.(1 point - 0.2/ each)
Each of the following sentences has an error. Find the errors and correct them.
46. We understand them as well as they understand we.
47. I am not sure where he’s from. He’s either Spanish nor Italian.
48. She said that she didn’t know what her father was alike because he died when she was still
young.
49. Before to go out, remember to turn off the lights.
50. Her sister knows how for prepare Chinese foods.
Part 4: Questions from 51 to 55.(1 point - 0.2/ each)
Fill in each of the sentences with correct prepositions.
51. Although he studied hard, he couldn't succeed …………… getting a high score in his test.
52. Most people agree that kindergarten contributes ………………the child's mental development.
53. I plan to keep up ………………my friend when we go running. I don't like to get behind him.
54. Sue's put in ………………the position of assistant teen reporter.
55. The dinosaurs died ………………millions of years ago.
SECTION III: READING (6.0 points)
Part 1. Questions 56 – 65 (2 points - 0.2/ each)
Read the passage below and choose the best answer to each question.
Choose the best answer to fill in each blank.
From the seeds themselves to the machinery, fertilizers and pesticides - The Green
Revolution regimen depend heavily on technology. One (56) ________, however, depends
much (57) ________ on technology - organic farming. Many organic farmers use machinery,
but (58) ________ chemical fertilizers or pesticides. (59) ________ chemical soil enrichers,
they use animal manure and plant parts not used as food -,natural,organic fertilizers that are
clearly a renewable (60) ________. Organic farmers also use alternatives (61) ________
pesticides; for example they may rely on natural predators of certain insect pests. (62)
________ the need arises, they can buy the eggs and larvae of these natural predators and
introduce them into their crop fields. They use (63) ________ techniques to control pests as
well, like planting certain crops together because one crop repels the other's pests. Organic
farmers do not need a lot of land; (64) ________ organic farming is perfectly (65) ________
to small farms and is relatively inexpensive. Finally, many organic farmers' average yields
compare favorably with other farmers' yields.
56. A. alteration B. alternate C. alternative D. alternation
57. A. more B. less C. better D. worse
58. A. also B. for C. not D. all
59. A. In .spite of B. On account of C. In favour of D. Instead of
60. A. resource B. source C. matter D. substance
61. A. of B. to C. for D. from
62. A. Then B. If C. Because D. Though
63. A. others B. another C. the others D. other
64. A. instead B. in one way C. on one hand D. in fact
65. A. suitable B. open C. likely D. suited
Part 2. Questions 66 – 75 (2.0 points - 0.2/ each)
Read the passage below and choose the best answer to each question
The rules of etiquette in American restaurants depend upon a number of factors the
physical location of the restaurant, e.g., rural or urban; the type of restaurant, e.g., informal
or formal; and certain standards that are more universal. In other words, some standards of
etiquette vary significantly while other standards apply almost anywhere. Learning the proper
etiquette in a particular type of restaurant in a particular area may sometimes require
instruction, but more commonly it simply requires sensitivity and experience. For example,
while it is acceptable to read a magazine in a coffee shop, it is inappropriate to do the same in
a more luxurious setting. And, if you are eating in a very rustic setting it may be fine to tuck
your napkin into your shirt, but if you are in a sophisticated urban restaurant this behavior
would demonstrate a lack of manners. It is safe to say, however, that in virtually every
restaurant it is unacceptable to indiscriminately throw your food on the floor. The conclusion
we can most likely draw from the above is that while the types and locations of restaurants
determine etiquette appropriate to them, some rules apply to all restaurants.
66. What topic is this passage primarily concerned with?
A. rules of etiquette B. instruction in proper etiquette
C. the importance of good manners D. variable and universal standards of etiquette
67. According to the passage, which of the following is a universal rule of etiquette?
A. tucking a napkin in your shirt B. not throwing food on the floor
C. reading a magazine at a coffee shop D. eating in rustic settings
68. According to the passage, ________ requires sensitivity and experience.
A. learning the proper etiquette B. asking for instructions
C. knowing the type of restaurant D. knowing about an area
69. The word "rustic" in the passage is closest in meaning to ________.
A. agricultural B. ancient C. unsophisticated D. urban
70. The word "tuck" in the passage is closest in meaning to ________.
A. put B. set C. hold D. fold
71. The word "sophisticated" in the passage could best be replaced by ________
A. expensive B. cultured C. famous D. exclusive
72. The author uses the phrase "safe to say" in order to demonstrate that the idea is
________.
A. somewhat innocent B. quite certain
C. very clever D. commonly reported
73. The word "indiscriminately" could best be replaced by ________.
A. randomly B. angrily C. noisily D. destructively
74. The word "draw" in the passage is closest in meaning to ________.
A. pick out B. drag away C. evoke D. infer
75. What is the author's main purpose in this passage?
A. to assist people in learning sophisticated manners
B. to describe variations in restaurant manners
C. to simplify rules of restaurant etiquette
D. to compare sophisticated and rustic restaurants
Part 3: Questions 76 – 85 (2.0 points - 0.2/ each)
Read the text and fill in one word which best fits each gap.
Our oceans are becoming extremely polluted. Most of this (76) ………………….……… comes from
the land, which means it comes from people. Firstly, there is raw (77) ………………….………, which is
pumped directly (78) ………………….……… the sea. Many countries, both developed and developing, are
guilty (79) ………………….……… doing this. Secondly, ships drop about 6 million tons of garbage into the
(80) ………………….……… each year. Thirdly, there are oil spills from ships. A ship has (81) ………………….………
accident and oil leaks from the vessel. This not (82) ………………….……… pollutes the water, but it also
kills marine life. Next, there (83) ………………….……… waste materials from factories. Without proper
regulations, factory owners let the waste run directly into the rivers, (84) ………………….……… then leads
to the sea. And finally, oil is washed from the land. This can be (85) ………………….……… result of
carelessness or a deliberate dumping of waste.
SECTION IV: WRITING (4.0 points)
Part 1: Questions from 86 to 90.(2 point- 0.4/ each)
Finish each of the following sentences in such a way that it means the same as the sentence
printed before it.
86. It is said that he escaped to a neutral country.
He
87. Although he was very tired, he agreed to play tennis.
Tired
88. “You have done really well to pass your driving test so quickly.” She said to me
She congratulated
89. I regret not going to the airport to say goodbye her.
I wish
90. As he grew older, he became more and more forgetful.
The older
Part 2: Questions from 91 to 92.(2 points - 0.4/ each)
Complete the second sentence so that it has a similar meaning to the first sentence, using
the word given. Do not change the word given.
91. You must never let anyone use your Identity Card. should
- Under ………………….…………………………………………….……… let anyone else use your identity card.
92. Many people believe that Edison has the ability to become world judo champion. capable
- Many people believe that Edison ………………….…………………………the world championship in judo.
93. My passport needs renewing because I´m going abroad this summer. get
- I need ………………….…………………………………………….……… because I´m going abroad this summer.
94. Clara said that she had not seen the missing letter. having
- Clara ………………….…………………………………………….……… the missing letter.
95. It's important to consider everyone's opinion before a final decision is made. account
- Everyone's opinion must ………………….……………………………….before a final decision is made.

……………………THE END ……………………...


- Thí sinh không được sử dụng tài liệu
- Giám thị không giải thích gì thêm
SỞ GIÁO DỤC VÀ ĐÀO TẠO KỲ THI CHỌN HỌC SINH GIỎI
CẤP TỈNH LỚP 9 NĂM 2018
(Đề thi gồm có 08 trang) MÔN: TIẾNG ANH
Thời gian làm bài: 120 phút (không kể thời gian phát đề)
Ngày thi: ……………………….

ĐỀ 7

SECTION I: LISTENING (3.0 points) (You will hear twice for each part)
Part 1. Listen to Adam talking to Carol about his day. Choose the correct answer.
1 Adam went to bed late because…
A. he was fixing his computer. B. he didn’t feel tired. C. his son felt sick.
2 Adam’s son returned to bed at…
A. four o’clock a.m. B. six o’clock a.m. C. a quarter to seven a.m.
3 At ten to eight,
A. Adam’s alarm clock went off. B. Adam got up. C. Adam left home.
4 In the morning, Adam didn’t have time to…
A. wash. B. have breakfast. C. buy petrol.
5 Adam borrowed money to buy…
A. a bus ticket. B. lunch. C. petrol.
6 This evening, Adam will…
A. relax and watch TV. B. spend time with his family. C. be very busy.
Part 2. A man needs to buy a part for his car. He calls a company to order the part. Listen
and complete the details in the order form.
Forename: Allan
Surname: (7) ……………………………………..
Postcode: (8) ……………………………………..
Postal address: 27 (9) …………………………Road, Nutley
Part Required: Inside Light
Make: (10) ……………………………………..
Model: Spirit
Year: (11) ……………………………………..
Cost Including Tax and
(12) £……………………………………..
Postage:
Card Number: (13) ……………………………………..
Expiry Date: (14) ……………………………………..
Security Code: (15) ……………………………………..

SECTION II: LEXICO – GRAMMAR (7.0 points)


Part 1: Questions from 16 to 36. (3.0 points - 0.15/ each)
Choose the best answer to each question.
16. Pick me up at 4 p.m. I _________ my bath by then.
A. will have B. will be having C. will have had D. must have had
17. She said that she would be punctual for the opening speech, ________ she were late?
A. but what if B. how about C. and what about D. so if
18. What are the main ________ of this illness?
A. traces B. emblems C. tokens D. symptoms
19. We believe that these animals could be saved if our plan were ________.
A. adopted B. taken up C. practised D. exploited
20. Local people are concerned about pollution from ________ oil wells.
A. maritime B. sea-going C. off-shore D. coast line
21. The topic ________ at yesterday’s meeting was of great importance.
A. to be discussed B. to have been discussed
C. discussed D. having been discussed
22. Evidence came up _________ specific speech sounds are recognized by babies as young as 6
months old.
A. what B. when C. which D. that
23. There have been protests from animal rights group about ________ on animals.
A. experience B. experiments C. expiration D. trials
24. She couldn’t perform well once she was _________.
A. under pressure B. out of mind C. in the mood D. over the moon
25. All the sales people I met always try to _________ buying their products.
A. persuade me for B. force me to C. talk me into D. help me with
26. There was nothing special about his clothes________ from his flowery tie.
A. but B. except C. other D. apart
27. She always________ the crossword in the paper before breakfast.
A. makes B. writes C. does D. works
28. When the tenants failed to pay their bill, the authorities decided to cut________ the gas
supply to the flat.
A. down B. out C. across D. off
29. His parents agreed to ________ him their car while they were away on holiday.
A. borrow B. lend C. hire D. let
30. We try to _________to see our parents at least twice a month.
A. call up B. go up C. come on D. drop in
31. He has always looked _________ his elder brother.
A. up to B. back on C. intoD. up and down
32. ________ what most people say about him, Paul has a very good sense of humour.
A. Opposite to B. Against C. Contrary to D. Opposing
33. They are happily married although, of course, they argue _________.
A. most times B. from day to day C. every now and then D. on the occasion
34. The water company will have to ______ off water supply while repairs to the pipes carry
out.
A. cut B. take C. break D. set
35. We could make better ________ of our energy resources.
A. spend B. use C. need D. limit
Part 2: Questions from 36 to 45. (2 points - 0.2/ each)
Use the correct form of the word in brackets to complete the following sentences.
36. This took place at the end of the …………………………………… century. TWENTY
37. People really …………………………………… the party. JOY
38. Meditation is great as a means of ……………………………………. RELAX
39. This method is a lot more …………………………………… than the previous one. EFFECT
40. Picasso was a Spanish …………………………………… who also lived in France. ART
41. There has been little …………………………………… in the negotiations since January. MOVE
42. …………………………………… enough, I didn't know she already had four children. SURPRISE
43. His voice was …………………………………… recognizable. INSTANT
44. The only cure for …………………………………… is creativity. BORE
45. This is a textile company …………………………………… in denim. SPECIAL
Part 3: Questions from 46 to 50.(1 point - 0.2/ each)
Each of the following sentences has an error. Find the errors and correct them.
46. A dog should be checked regular by a reterinarian to ensure that it remains in good health.
47. George has not completed the assignment yet , and Mary hasn’t neither.
48. Although my sister was tired, but she helped me with my homework.
49. Sinaitic is the name of an alphabet which developed their symbols from Egyptian hieroglyphics.
50. There are many different ways of comparing the economy of one nation with those of another.
Part 4: Questions from 51 to 55.(1 point - 0.2/ each)
Fill in each of the sentences with correct prepositions.
51. Did you know that Paul is married ……………. Susie last week?
52. According ……………. the timetable, the next train is due in two hours.
53. Apart ……………. you, nobody else knows that I have escaped from prison.
54. Please go ……………. your homework and look for the mistakes once more time.
55. Volunteers were handing ……………. food and clothes after the flood in the mountainous area.
SECTION III: READING (6.0 points)
Part 1. Questions 56 – 65 (2 points - 0.2/ each)
Read the passage below and choose the best answer to each question.
Nowadays people are more aware that wildlife all over the world is in (56)
________many species of animals are threatened, and could easily become (57) ________ if
we do not make an effort to (58) ________them. There are many reasons for this. In some
cases, animals are (59) ________for their fur or for other valuable parts of their bodies. Some
birds, such as parrots, are caught (60) ________, and sold as pets. For many animals and
birds, the problem is that their habitat- the place where they live- is (61) ________. More (62)
________is used for farms, for houses and industry , and there are fewer open spaces than
there once were. Farmers use powerful chemicals to help them grow better (63) ________,
but these chemicals pollute the environment and (64) ________wildlife. The most successful
animals on earth- human beings- will soon be the only ones (65) ________, unless we can
solve this problem.
56. A. problem B. vanishing C. threat D. danger
57. A. empty B. disappeared C. extinct D. vanished
58. A. serve B. protect C. defend D. harm
59. A. game B. extinct C. chased D. hunted
60. A. alive B. for living C. lively D. for life
61. A. departing B. exhausting C. disappearing D. escaping
62. A. land B. area C. soil D. earth
63. A. products B. fields C. crops D. herbs
64. A. wound B. wrong C. harm D. spoil
65. A. staying B. survived C. over D. left
Part 2. Questions 66 – 75 (2.0 points - 0.2/ each)
Read the passage below and choose the best answer to each question
Early peoples had no need of engineering works to supply their water. Hunters and
nomads camped near natural sources of fresh water, and populations were so sparse that
pollution of the water supply was not a serious problem. After community life developed and
agricultural villages became urban centres, the problem of supplying water became important
for inhabitants of a city, as well as for irrigation of the farms surrounding the city. Irrigation
works were known in prehistoric times, and before 2000 BC the rulers of Babylonia and Egypt
constructed systems of dams and canals to impound the flood waters of the Euphrates and
Nile rivers, controlling floods and providing irrigation water throughout the dry season. Such
irrigation canals also supplied water for domestic purposes. The first people to consider the
sanitation of their water supply were the ancient Romans, who constructed a vast system of
aqueducts to bring the clean waters of the Apennine Mountains into the city and built basins
and filters along these mains to ensure the clarity of the water. The construction of such
extensive water-supply systems declined when the Roman Empire disintegrated, and for
several centuries local springs and wells formed the main source of domestic and industrial
water.
The invention of the force pump in England in the middle of the 16th century greatly
extended the possibilities of development of water-supply systems. In London, the first
pumping waterworks was completed in 1562; it pumped river water to a reservoir about 37
m above the levelof the River Thames and from the reservoir the water was distributed by
gravity, through lead pipes, to buildings in the vicinity.
Increased per-capita demand has coincided with water shortages in many countries.
Southeast England, for example, receives only 14 per cent of Britain's rainfall, has 30 per cent
of its population, and has experienced declining winter rainfall since the 1980s.
In recent years a great deal of interest has been shown in the conversion of seawater to
fresh water to provide drinking water for very dry areas, such as the Middle East. Several
different processes, including distillation, electrodialysis, reverse osmosis, and direct-freeze
evaporation, have been developed for this purpose. Some of these processes have been used
in large facilities in the United States. Although these processes are successful, the cost of
treating seawater is much higher than that for treating fresh water.
66. Early peoples didn’t need water supply engineering works because ______.
A. their community life had already developed
B. natural sources of fresh water nearby were always available
C. there was almost no dry season in prehistoric times
D. they had good ways to irrigate their farms
67. The word “impound” in paragraph 1 is closest in meaning to “______”.
A. supply B. irrigate C. provide D. drain
68. Clean water supply was first taken into consideration by ______.
A. the English people B. the ancient Romans
C. the Egyptians D. the US people
69. For several centuries after the disintegration of the Roman Empire, the main source of
water supply was from ______.
A. springs and wells B. systems of aqueducts
C. dams and canals D. water pipes
70. The word “mains” in paragraph 1 could best be replaced by “______”.
A. lands B. areas C. pipes D. rivers
71. Which of the following is NOT true about London’s water supply in the middle of the 16th
century?
A.Water was pumped from the River Thames. B. Water was stored in a reservoir.
C. Water ran from the reservoir to buildings. D. Water was conducted through canals.
72. The word “vicinity” in paragraph 2 refers to ______.
A. the cities in South-east England B. the areas along the River Thames
C. the neighborhood around a reservoir D. the region where industry developed
73. One of the causes of water shortages in South-east England is ______.
A. water pollution B. increased demand
C. water-supply system decline D. water evaporation
74. Which of the following is NOT mentioned as a process of conversing seawater to
freshwater?
A. Steaming and cooling B. Water evaporation
C. Dissolving chemicals D. Purification method
75. In the passage, the author mainly discusses ______.
A. the development of water supplyB. the results of water shortages
C. the water pumping systems D. the fresh water storage
Part 3: Questions 76 – 85 (2.0 points - 0.2/ each)
Read the text and fill in one word which best fits each gap.
For a long time in the past photography was not regarded (76) ………………………….. an art. It was
simply a skill and it was criticized for being too mechanical and not creative enough. At last,
however, photography is now accepted as a unique (77) ………………………….. very important form of
art.
The photograph's claim to be (78) ………………………….. objective record of reality is now seriously
challenged, and the important function of photography in modern-day society is consequently
(79) ………………………….. threat. The threat has suddenly become all the more serious as more and
more photographers are turning to the new technology offer (80) ………………………….. computers offer.
Moreover, an increasing (81) ………………………….. of colleges have now begun to courses in computer
imaging. All these developments raise a disturbing question. Is photography, as we know it
dead?
In (82) ………………………….. of its complete transformation by new technological developments,
however, photography will continue to play a key (83) ………………………….. in our culture. Although it
may (84) ………………………….. longer claim to be realistic, modern photography can continue to provide
us (85) ………………………….. fresh visual information about ourselves and the world in which we live.
SECTION IV: WRITING (4.0 points)
Part 1: Questions from 86 to 90.(2 point- 0.4/ each)
Finish each of the following sentences in such a way that it means the same as the sentence
printed before it.
86. Peter hasn’t had his hair cut for over 3 months.
 It is over
87. People will only become aware of the problem if you increase the publicity.
 Only if
88. Scientists have tried very hard to find a cure for AIDS.
 Enormous
89. Success depends on hard work.
 The harder
90. If you changed your mind, you would be welcomed to join our class.
 Were you
Part 2: Questions from 91 to 92.(2 points - 0.4/ each)
Complete the second sentence so that it has a similar meaning to the first sentence, using
the word given. Do not change the word given.
91. He had to give the money back. obliged
He …………………….……………………………………………….. give the money back.
92. I didn’t need any help to repair my bicycle. repaired
I…………………….……………………………………………….. own.
93. Teachers from the British Council started this school. set
This school …………………….……………………………… teachers from the British Council.
94. ‘I won’t be able to finish the assignment,’ she said. explained
She …………………….……………………………………………….. be able to finish the assignment.
95. I thought I would get the job, but someone else got it instead. succeed
I …………………….……………………………………………….. the job.

……………………THE END ……………………...


- Thí sinh không được sử dụng tài liệu
- Giám thị không giải thích gì thêm

SỞ GIÁO DỤC VÀ ĐÀO TẠO KỲ THI CHỌN HỌC SINH GIỎI


CẤP TỈNH LỚP 9 NĂM 2018
(Đề thi gồm có 08 trang) MÔN: TIẾNG ANH
Thời gian làm bài: 120 phút (không kể thời gian phát đề)
Ngày thi: ……………………….

ĐỀ 8

SECTION I: LISTENING (3.0 points) (You will hear twice for each part)
Part 1. Listen to the recording twice. Choose correct answer for each of the following
questions.
1. What one thing does the girl NOT have to do on Wednesday after school?
A. practice the piano B. take care of children C. finish homework assignments
2. Why can't the girl go to a movie on Monday?
A. She has to catch up on her French homework.
B. She needs to write a paper.
C. She must practice for a math test.
3. How long is her soccer practice on Tuesday?
A. one hour B. an hour and a half C. two hours
4. What chore does the girl have to do on Saturday?
A. clean the garage B. pick up her roomC. finish her science project
5. Which movie showing is the girl going to see?
A. 5:00 p.m. B. 7:15 p.m. C. 9:00 p.m.
Part 2. Complete the form below. Write no more than two words or a number for each
answer.

TAUBER INSURANCE Co
Example Answer
Insurance type: Vehicle
Policy: (6) …………………………………
Make & Model: Masda Marvel
Engine size: (7) …………………………………cc
Name: Lisa Marie Heathcote
Date of Birth: (8) …………………………………1955
Password: (9) …………………………………
Change valuation? Yes
No
Reduce value to: (10) $ …………………………………
Notes to be added to policy:
Add new driver:
Name: Samel Michaels
Age: 28
Relationship to main driver: (11) …………………………………
Reason: (12) …………………………………
Client / new driver to provide:
* Verified (13) …………………………………of driver licence
* Clean driving (14) …………………………………
Start date:
* (15) …………………………………
* Full cover when paperwork appoved
SECTION II: LEXICO – GRAMMAR (7.0 points)
Part 1: Questions from 16 to 36. (3.0 points - 0.15/ each)
Choose the best answer to each question.
16. Some people show_________ attitude toward the misery of others, totally untouched by
their suffering.
A. passionate B. dispassionate C. passion D. passionately
17. I enjoy walking to school, but on rainy days I_________ to going by bus.
A. would rather B. resort C. commit D. prefer
18. This is a photograph of the school I_________ when I lived in Swindow.
A. went B. studied C. attended D. joined
19. The train was_________ by a heavy snowfall.
A. held out B. held off C. held up D. held back
20. The people in my class, _________ are very friendly.
A. most of international students B. the most international students
C. almost international students D. mostly international students
21. Doctors advise people who are deficient_________ Vitamin C to eat more fruit and
vegetables.
A. from B. for C. of D. in
22. The teacher made a difficult question, but at last, Joe_________ a good answer.
A. came up against B. came up with C. came up to D. came up for
23. _________ at his lessons, still he couldn't catch up with his classmates.
A. Hard as he was B. Hard as he does C. Hardly as he worked D. Hard as he worked
24. He refused to give up work, _________ he had won a million dollars.
A. despite B. even though C. However D. as though
25. Jane wasn't in when I arrived. I suppose she_________ I was coming.
A. may forget B. can't have forgotten C. must forget D. must have forgotten
26. Not for a moment_________ the truth of the case.
A. he doubted B. was he doubting C. he did doubt D. did he doubt
27. The student_________ the highest score will be awarded a scholarship.
A. receives B. who is receiving C. who receive D. receiving
28. On leaving prison, Vic decided to turn over a new_________ and to give up his old life of
crime.
A. leafB. book C. chapter D. page
29. He's been very sick. His doctor insisted that he_________ in bed this week.
A. stays B. stay C. would stay D. will stay
30. According to some historians, if Napoleon had not invaded Russia, he_________ the rest
of the world.
A. would have conquered B. conquered
C. had conquered D. would conquer
31. He was so mean that he could not bear to_________ the smallest sum of money for the
charity appeal.
A. part with B. let out C. give in D. pay off
32. We_________ for three hours and are very tired.
A. have been walking B. are walking C. were walking D. had been walking
33. _________ weather! We can't go out for a walk now.
A. What a terrible B. How a terrible C. What terrible D. How terrible
34. Susan: "How much do you earn, Joe?" Joe: “I'd ________________".
A. rather don't say B. rather not say C. better not to say D. prefer not say
35. Anna: "You don't have to go to school this afternoon, do you?" Bob:
“_____________”.
A. No, I needn't B. What does that mean?
C. Yes, I don't have to D. That's OK
Part 2: Questions from 36 to 45. (2 points - 0.2/ each)
Use the correct form of the word in brackets to complete the following sentences.
36. The teacher asked us to …………………………………the main ideas in the chapter we had SUMMARY
read in class.
37. The top manager of the shop told me that my credit card was not VALIDATE
…………………………………to pay for the jeans which cost under 20€.
38. Art in the 20th century is usually referred to as …………………………………art. MODERNIZE
39. Everybody listens to David Guetta's music. It's so …………………………………. COMMERCE
40. Butterflies are thoroughly …………………………………by scientists. CLASS
41. She …………………………………the qualities of a good leader. EXAMPLE
42. Many people don't understand the …………………………………of life. SIMPLE
43. It's impossible to tell those twins apart! They are …………………………………. IDENTITY
44. He made several wrong …………………………………about women. GENERAL
45. It was hard to …………………………………between the two styles of music. DIFFERENCE
Part 3: Questions from 46 to 50.(1 point - 0.2/ each)
Each of the following sentences has an error. Find the errors and correct them.
46. He said he must come to the party if he finishes his assignment for next week's seminar.
47. The basic law of addition, subtraction, multiplication and division are taught to all elementary
school students.
48. Sandra has not rarely missed a play or concert since she was seventeen years old.
49. Many of the famous advertising offices are located in Madison Avenue.
50. Each of the nurses report to the operating room when his or her name is called.
Part 4: Questions from 51 to 55.(1 point - 0.2/ each)
Fill in each of the sentences with correct prepositions.
51. Graduates are well equipped …………………careers in environmental policy.
52. The course encourages students to think critically …………………development.
53. This course provides the opportunity to focus …………………your major area of interest.
54. The young reporter looked up …………………Ken, who'd been a journalist for thirty years.
55. I never wanted to go along …………………their ideas, but I had to in the end.
SECTION III: READING (6.0 points) giaoa andethitie ng anh.i nfo
Part 1. Questions 56 – 65 (2 points - 0.2/ each)
Read the passage below and choose the best answer to each question.
Most people feel that when they dream , they are (56) ________ off to another world.
On the (57) ________, dreams are often connected to our daily lives. When our whole (58)
________ is filled with something, when we are either very upset (59) ________ when we
are in good spirit, a dream will represent this reality in symbols. It is often (60) ________ that
we benefit from dreams because they help the spirit to heal itself, when things (61) ________
wrong. Dreams are therefore a kind of escape, almost a holiday from (62) ________ life, with
its fears and responsibilities. It is, however, a strange kind of holiday because whether we
have a wonderful time or whether it turns (63) ________ to be a nightmare, we quickly forget
it. Most dreams disappear forever, (64) ________ you are one of those people disciplined
enough to write them down as soon as you (65) ________.
56. A. taken B. guided C. brought D. carried
57. A. contrast B. contrary C. other side D. opposite
58. A. spirit B. mind C. brain D. soul
59. A. unless B. but also C. or else D. or
60. A. said B. spoken C. declared D. started
61. A. become B. go C. turn D. get
62. A. realB. factual C. genuine D. actual
63. A. off B. up C. out D. down
64. A. except that B. therefore C. thus D. unless
65. A. wake up B. arise C. awake D. rise up
Part 2. Questions 66 – 75 (2.0 points - 0.2/ each)
Read the passage below and choose the best answer to each question
Sleep is a natural process, and although a lot have been written about the subject, it is
still surrounded by mystery. It is used by some as an escape from the world, and regarded by
others as an irritating waste of time: some people get by on very little, others claim they
cannot exist without at least ten hours, but nobody can do without sleep completely.
Our night’s sleep does not just consist of a steady phase of gradually deepening sleep.
It alternates between two stages: Non-dreaming or ordinary sleep, and REM (rapid eye
movement) or dreaming sleep. As soon as we fall asleep, we go straight into non-dreaming
sleep for an hour or so, then into REM sleep for about 15 minutes, then back into non-
dreaming sleep. It alternates in this way for the rest of the night, with non-dreaming sleep
tending to last longer at the beginning of the night. Non-dreaming sleep occupies three-
quarters of our night’s sleep, about a quarter of it deep and the rest fairly light.
It is widely believed that sleep repairs the body and makes good the damage caused by
being awake. However, its main function is to refresh the brain. Experts believe that probably
only about two-thirds of our sleep is necessary for repairing and refreshing the brain, with the
most valuable sleep coming in the first few hours of the non-dreaming period, the last few
hours of sleep are not so essential. The brain can manage quite well with reduced sleep as
long as it is uninterrupted sleep.
The quality of sleep is important. A study conducted in the USA looked at short
sleepers, who slept for 5.5 hours on average, and long sleepers, who had 8.5 hours or more. It
is discovered after a variety of tests that the long sleepers were poor sleepers, had twice as
much REM sleep as the short sleepers, appeared to sleep longer to make up for poor sleep,
and did not wake up in the morning refreshed. Similarly, people who sleep deeply do not
necessarily get a better quality of sleep than shallow sleepers. Deep sleepers can feel tired the
following day, so six hours of good sleep is worth more than eight hours of troubled sleep.
66. It can be concluded from the first paragraph that ______.
A. people need equal time of sleep B. sleep remains a puzzle
C. sleep is among the processes of the nature D. everything about sleep has been brought
to light
67. The word “irritating” in paragraph 1 is closest in meaning to ______.
A. calming B. soothing C. annoying D. comforting
68. All the following statements are true, EXCEPT for ______.
A. our night’s sleep occurs in a straight line of only two phases
B. all sleeps are similar in the alternatives of the two stages during the night
C. we spend only 25 percent of our night’s sleeping time dreaming
D. we often have no dreams right after we fall asleep
69. The word “it” in paragraph 2 refers to ______.
A. our night’s sleep B. the ordinary sleep C. the REM D. the night
70. The word “occupies” in paragraph 2 could be best replaced by ______.
A. accounts for B. takes care of C. works out D. goes up
71. Unlike the common belief, sleep helps ______.
A. not to be awake C. us to fix the damage happening by day
B. us to repair our body D. our brain to rest and recover
72. The study discussed in the reading passage suggests that ______.
A. the fewer hours we sleep, the more we dream
B. deep sleep means better sleep
C. the type of sleep is more important than its length
D. six hours of sleep is better than eight hours
73. Which of the following is NOT discussed in the passage?
A. The role of the sleep. B. Sleepless people’s problems.
C. Types of sleep. D. The circle of a sleep.
74. It can be inferred from the reading passage that ______.
A. if we can sleep uninterruptedly, it is not necessary to sleep the whole night
B. REM makes good our brain
C. nearly 70 % of our sleep is invaluable
D. dream enables our body to refresh when we can sleep uninterruptedly
75. This passage is the most likely taken from ______.
A. a doctor’s description B. a health magazine
C. an advertisement D. a fashion magazine
Part 3: Questions 76 – 85 (2.0 points - 0.2/ each)
Read the text and fill in one word which best fits each gap.
Doctors have reported that vitamins (76) ………………………… substances required for the proper
functioning of the body. In this century, thirteen vitamins have been discovered. A lack of any
vitamins in a person’s body can (77) ………………………… diseases. In some cases, an excess of vitamins
can also lead to illness. For (78) …………………………, sailors in the past were prone to suffer from scurvy
that is a disease resulting from the lack of vitamin C. It causes bleeding of the gum, loss of
teeth and skin rashes. Sailors suffer (79) ………………………… scurvy because they did not eat (80)
………………………… and vegetables. Fruits and vegetables contain vitamin C which is necessary for good
health.
Vitamin B complex is (81) ………………………… of eight different vitamins. A lack of any of these
vitamins will lead to different diseases. For instance, a person who has (82) ………………………… little
vitamin B1 will suffer from beriberi, a disease that causes heart problems and mental
disorders. A lack of vitamin B2 results (83) ………………………… eye and skin problems while deficiency of
vitamin B6 causes problems of the nervous system. Too little vitamin B12 will cause anemia.
The knowledge (84) ………………………… vitamin deficiencies caused certain diseases led doctors to cure
people suffering from these illnesses by giving them doses of the necessary vitamins. Today,
vitamins are available in the form of pills and can easily (85) ………………………… bought at any
pharmacy.
SECTION IV: WRITING (4.0 points)
Part 1: Questions from 86 to 90.(2 point- 0.4/ each)
Finish each of the following sentences in such a way that it means the same as the sentence
printed before it.
86. Don’t you think we should ask the price?
Hadn’t
?
87. Doris tiptoed up the stairs because she didn’t want to wake anyone up.
To
88. I’m sure it wasn’t Mr. Bill you saw because he’s in New York.
It can’t
89. The hurricane blew the roof off the house.
The house had
90. I really think she should stop ignoring me in meetings
It's about
90. Pamela isn't sorry she decided to move to Florida.
Pamela has
Part 2: Questions from 91 to 92.(2 points - 0.4/ each)
Complete the second sentence so that it has a similar meaning to the first sentence, using
the word given. Do not change the word given.
91. They offered her a job but she refused. turned
She ……………………………………………………………………………… of a job.
92. They find driving on the left in England very strange. accustomed
They ……………………………………………………………………………… driving on the left in England.
93. I expect you were very tired by the end of the week. must
By the end of the week …………………………………………………………………………………………………………………………………………… exhausted.
94. Being late is inexcusable. excuse
There is ……………………………………………………………………………………………………………………………………………………… late.
95. Matthew decided not to do his homework and went to play football. instead
Matthew went to play football …………………………………………………………………………………………………………………………………..………
homework.

……………………THE END ……………………...


- Thí sinh không được sử dụng tài liệu
- Giám thị không giải thích gì them
SỞ GIÁO DỤC VÀ ĐÀO TẠO KỲ THI CHỌN HỌC SINH GIỎI
CẤP TỈNH LỚP 9 NĂM 2018
MÔN: TIẾNG ANH
(Đề thi gồm có 08 trang) Thời gian làm bài: 120 phút (không kể thời gian phát đề)
Ngày thi: ……………………….

ĐỀ 9

SECTION I: LISTENING (3.0 points) (You will hear twice for each part)
Part 1. Listen to the recording twice. Choose the correct answer for each of following
questions.
1. The woman __________.
A. has gone skiing many times B. doesn't have skiing experience C. skis once to twice
a year
2. The ski slopes are ____________.
A. very crowdedB. somewhat icy C. nearly empty
3. Another skier ___________.
A. drove into the woman B. forced her into the trees C. stole her new purple
gloves
4. When the woman finds the other skier, she is going to __________.
A. break his face, arm, and legs
B. show his face on social media
C. hit the person in the head with a book
5. The woman says she can recognize the other skier by his __________.
A. hair and eyes B. crazy laugh C. clothing
Part 2. Listen to the recording twice. Complete the form below. Write no more than two
words or numbers for each answer.
Example Answer
Purpose placing an advertisement.
Laptop FOR Sale
Condition: Almost new
Weight: (6) …………………………….
Make:Allegro
Memory: (7) …………………………….GB
Sceen: (8) …………………………….cm
Touch pad but with mouse
Number of ports: Two
Battery lasts: (9) …………………………….
Latest programmes: Not (10) …………………………….
Extras:
Web cam
Printer with:(11) ……………………….and …………………………
Smart case:
Price: (12) …………………………….
Contact details
Name: David (13) …………………………….
E-mail address: DIB_7791@hotmail.com
Mobile number: (14) …………………………….
Advert placed: (15) …………………………….
SECTION II: LEXICO – GRAMMAR (7.0 points)
Part 1: Questions from 16 to 36. (3.0 points - 0.15/ each)
Choose the best answer to each question.
16. You find it hard to _____ your temper if you think someone is making a fool out of you.
A. cool B. lose C. recover D. keep
17. Remember to _____ your shoes when you are in a Japanese house.
A. take on B. take off C. take up D. put on
18. I have _____ information about her situation to tell you.
A. too few B. so many C. so little D. too little
19. America, as well as Japan, England, and Germany_____.
A. are developed countries B. are developing countries
C. is a developed country D. is developing countries
20. It's no good _____ your father about your failure.
A. to tell B. tell C. telling D. told
21. I’m afraid I don’t _____ your view on this matter, but let’s not quarrel about it.
A. correspond B. equate C. accord D. share
22. He got an excellent grade in his examination_____ the fact that he had not worked
particularly hard.
A. on account of B. because C. in spite of D. although
23. He always _____ the crossword in the newspaper before breakfast.
A. writes B. makes C. works D. does
24. He just couldn't open the jar _____ hard he tried.
A. however B. whatever C. moreover D. even
25. “Today’s my 20th birthday.” - “______”
A. Take care! B. Many happy returns!
C. Have a good time! D. I don’t understand.
26. Tom: How are things going? Mary: _____.
A. I can't complain B. There are a lot of things
C. Not much D. That’s all right.
27. Vietnam's rice export this year will decrease ______ about 10%, compared with that of
last year.
A. with B. at C. by D. on
28. I’d like to contribute_____ the school Red Cross fund.
A. on B. to C. with D. for
29. “In spite of her deafness, she plays the violin very well.” Deafness means ______.
A. inability to speak B. inability to see
C. inability to hear D. mentally impairment
30. I am going to have my eye_____ tomorrow.
A. to test B. testC. testing D. tested
31. A: Which one do you like? B: I like ______ one of the two.
A. the bigger B. biggest C. bigger D. the biggest
32. Robert and his wife _____ to my house for tea yesterday evening.
A. came about B. came down C. came round D. came away
33. Each of the guests _____ a bunch of flowers.
A. is given B. are given C. give D. were given
34. We should arrive home safe and _____.
A. sound B. soundly C. warm D. warmly
35. The idea got a lot of _____ from the children’s parents.
A. opposition B. disagreement C. dislike D. denial
Part 2: Questions from 36 to 45. (2 points - 0.2/ each)
Use the correct form of the word in brackets to complete the following sentences.
36. …………………………is a part of life, but you have to be able to enjoy the others too. SAD
37. Without that, I wouldn't be happy even if I was the …………………………man in the RICH
world.
38. The music in the festival was so loud. It was …………………………! DEAF
39. I don't think my marks can get …………………………because I already study to the best of HIGH
my abilities.
40. The …………………………river fish in Europe lives in Spain! LARGE
41. Passeig de Gràcia is being …………………………so there is more room for people to walk WIDE
on the pavement.
42. I can …………………………that if you study you will have no problems passing this test. SURE
43. Nowadays there are lots of …………………………species of animals in the world, for DANGER
example, the panda.
44. I was …………………………to take the driving test by my friends. COURAGE
45. I see they've finally got round to …………………………the Shoreham road. WIDE
Part 3: Questions from 46 to 50.(1 point - 0.2/ each) Giaoan dethitienganh.info

Each of the following sentences has an error. Find the errors and correct them.
46. Nuclear powers production in the US is controlled by the Nuclear Regulatory Commission
[NRC].
47. All of we students must have an identification card in order to check books out of the
library.
48. A fiber-optic cable across the Pacific went into service in April 1989, link the United States and
Japan.
49. A good artist like a good engineer learns as much from their mistakes as from successes.
50. Almost all life depends to chemical reactions with oxygen to produce energy.
Part 4: Questions from 51 to 55.(1 point - 0.2/ each)
Fill in each of the sentences with correct prepositions.
51. He is always getting …………………trouble because of his carelessness.
52. You cross a cheque …………………drawing two lines in it and writing & co like this.
53. Not many old people approve …………………the ways of the younger people.
54. The local people look down …………………anyone who throws litter around their lovely park.
55. The editor never puts up …………………articles that they are boring.
SECTION III: READING (6.0 points)
Part 1. Questions 56 – 65 (2 points - 0.2/ each)
Read the passage below and choose the best answer to each question.
The ability to weep is a uniquely human form of emotional response. Some scientists
have suggested that human tears are (56) _____ of an aquatic past – but this does not seem
very likely. We cry from the moment we enter this world, for a number of reasons. Helpless
babies cry to persuade their parents that they are ill, hungry or uncomfortable. As they (57)
_____, they will also cry just to attract parental attention and will often stop when they get it.
The idea that having a good cry do you (58) _____ is a very old one and now it has
scientific validity since recent research into tears has shown that they (59) _____ a natural
painkiller called enkaphalin. By fighting sorrow and pain this chemical helps you feel better.
Weeping can increase the quantities of enkaphalin you (60) _____.
Unfortunately, in our society we impose restrictions upon this naturally (61) _____ activity.
Because some people still regard it as a (62) _____ of weakness in men, boys in particular are
admonished when they cry. This kind of repression can only increase stress, both emotionally
and physically.
Tears of emotion also help the body (63) _____ itself of toxic chemical waste, for there
is more protein in them than in tears resulting from cold winds or other irritants. Crying
comforts, calms and can be very enjoyable – (64) _____ the popularity of highly emotional
films which are commonly (65) _____ “weepies”. It seems that people enjoy crying together
almost as much as laughing together.
56. A. witness B. evidence C. result D. display
57. A. evolveB. change C. develop D. alter
58. A. better B. fineC. good D. well
59. A. contain B. retain C. hold D. keep
60. A. construct B. achieve C. provide D. produce
61. A. curing B. treating C. healing D. improving
62. A. hint B. symbol C. feature D. sign
63. A. release B. rid C. loosen D. expel
64. A. consider B. remark C. distinguish D. regard
65. A. named B. entitled C. subtitled D. called
Part 2. Questions 66 – 75 (2.0 points - 0.2/ each)
Read the passage below and choose the best answer to each question
By the mid-nineteenth century, the term "icebox" had entered the American language,
but ice was still only beginning to affect the diet of ordinary citizens in the United States. The
ice trade grew with the growth of cities. Ice was used in hotels, taverns, and hospitals, and by
some forward-looking city dealers in fresh meat, fresh fish, and butter. After the Civil
War(1861-1865), as ice was used to refrigerate freight cars, it also came into household use.
Even before 1880, half the ice sold in New York, Philadelphia, and Baltimore, and one-third of
that sold in Boston and Chicago, went to families for their own use. This had become possible
because a new household convenience, the icebox, a precursor of the modern refrigerator,
had been invented. Making an efficient ice box was not as easy as we might now suppose. In
the early nineteenth century, the knowledge of the physics of heat, which was essential to a
science of refrigeration, was rudimentary. The commonsense notion that the best icebox was
one that prevented the ice from melting was of course mistaken, for it was the melting of the
ice that performed the cooling. Nevertheless, early efforts to economize ice included
wrapping the ice in blankets, which kept the ice from doing its job. Not until near the end of
the nineteenth century did inventors achieve the delicate balance of insulation and circulation
needed for an efficient icebox.
But as early as 1803, an ingenious Maryland farmer, Thomas Moore, had been on the right
track. He owned a farm about twenty miles outside the city of Washington, forwhich the
village of Georgetown was the market center. When he used an icebox of his own design to
transport his butter to market, he found that customers would pass up the rapidly melting
stuff in the tubs of his competitors to pay a premium price for his butter, still fresh and hard in
neat, one-pound bricks. One advantage of his icebox, Moore explained, was that farmers
would no longer have to travel to market at night in order to keep their produce cool.

66. What does the passage mainly discuss?


A. The influence of ice on the diet B. The development of refrigeration
C. The transportation of goods to market D. Sources of ice in the 19th century
67. According to the passage, when did the word "icebox" become part of the language of the
United States?
A. In 1803 B. Sometime before 1850
C. During the Civil War D. Near the end of the nineteenth century
68. The phrase "forward-looking" in line 3 is closest in meaning to ________.
A. progressive B. popular C. thrifty D. well-established
69. The author mentions fish in line 5 because ________.
A. many fish dealers also sold ice
B. fish was shipped in refrigerated freight cars
C. fish dealers were among the early commercial users of ice
D. fish was not part of the ordinary person's diet before the invention of the icebox
70. The word "it" in line 5 refers to ________.
A. fresh meat B. the Civil War C. ice D. a refrigerator
71. According to the passage, which of the following was an obstacle to the development of
the icebox?
A. Competition among the owners of refrigerated freight cars
B. The lack of a network for the distribution of ice
C. The use of insufficient insulation
D. Inadequate understanding of physics
72. The word "rudimentary" in line 10 is closest in meaning to ________.
A. growing B. undeveloped C. necessary D. uninteresting
73. According to the information in the second paragraph, an ideal icebox would ________.
A. completely prevent ice from melting B. stop air from circulating
C. allow ice to melt slowly D. use blankets to conserve ice
74. The author describes Thomas Moore as having been "on the right track" to indicate that
________.
A. the road to the market passed close to Moore's farm
B. Moore was an honest merchant
C. Moore was a prosperous farmer
D. Moore's design was fairly successful
75. According to the passage, Moore's icebox allowed him to ________.
A. charge more for his butter B. travel to market at night
C. manufacture butter more quickly D. produce ice all year round
Part 3: Questions 76 – 85 (2.0 points - 0.2/ each)
Read the text and fill in one word which best fits each gap.
THE BIRTH OF THE T-SHIRT
The T-shirt, or at least the T-shirt as we know it, was born in the theatre. When
Tennessee William's play A Streetcar Named Oesire opened in New York in December 1947, a
young actor (76) ………………… Marlon Brando went (77) …………………stage wearing a (78) …………………of blue
jeans and a bright, white, capped-sleeve T-shirt. It was the first time the T-shirt had (79)
…………………seen publicly as anything except an item of underwear and it set a fashion trend (80)

…………………was to last through until the end of the century. The idea for the T-shirt came from
Brando himself. He had worn one at rehearsals for the play. The director was (81) so
impressed by the look that was created that he asked Brando to wear the shirt in the play
itself. Brando may have seen the shirt being advertised (82) …………………the American company
Sears Roebuck. They had decided to market the shirt as a fashionable garment in its own
right, rather (83) …………………just something to be worn (84) …………………warmth beneath a denim
workshirt (85) …………………an army uniform. It was Brando. However, who popularized it, especially
with the release of the film version of Streetcar in 1951. A short leather jacket completed the
look that was to be adopted by teenage rebels in many countries for decades afterwards.
SECTION IV: WRITING (4.0 points)
Part 1: Questions from 86 to 90.(2 point- 0.4/ each)
Finish each of the following sentences in such a way that it means the same as the sentence
printed before it.
86. I'm sure they didn't plan this project.
They can't
87. People say he was dismissed last week.
He
88. "I'm sorry I missed the appointment with the doctor this morning." he said.
He apologised
89. Why don't you do your homework before going to class?
If I
90. This motorbike is cheaper than I thought I would be.
This motorbike is not
Part 2: Questions from 91 to 92.(2 points - 0.4/ each)
Complete the second sentence so that it has a similar meaning to the first sentence, using
the word given. Do not change the word given.
91. Without the help that Joe gave me, I don't think I would have finished the course. been
If it ……………………………………….………………….., I don't think I would have finished the course.
92. Susan doesn't intend to climb that mountain again. has
Susan ……………………………………….………………….. that mountain again.
93. After a long hard journey, I cheered up when I saw my home again. sight
After a long hard journey, I cheered up ……………………………………….………………….. of my home again.
94. I promised him that the situation would not be repeated in the future. word
I ……………………………………….…………… that there would be no repetition of the situation in the future.
95. Dario was just about to leave for the theatre when the phone rang. point
Dario ……………………………………….………………….. for the theatre when the phone rang.
……………………THE END ……………………...
- Thí sinh không được sử dụng tài liệu
- Giám thị không giải thích gì thêm
SỞ GIÁO DỤC VÀ ĐÀO TẠO KỲ THI CHỌN HỌC SINH GIỎI
CẤP TỈNH LỚP 9 NĂM 2018
MÔN: TIẾNG ANH
(Đề thi gồm có 08 trang) Thời gian làm bài: 120 phút (không kể thời gian phát đề)
Ngày thi: ……………………….

Đề 10

SECTION I: LISTENING (3.0 points) (You will hear twice for each part)

Part 1. Listen to the recording twice. Choose the correct answer for each of following
questions.
1. The woman __________.
A. has gone skiing many times B. doesn't have skiing experience C. skis once to twice
a year
2. The ski slopes are ____________.
A. very crowdedB. somewhat icy C. nearly empty
3. Another skier ___________.
A. drove into the woman B. forced her into the trees C. stole her new purple
gloves
4. When the woman finds the other skier, she is going to __________.
A. break his face, arm, and legs
B. show his face on social media
C. hit the person in the head with a book
5. The woman says she can recognize the other skier by his __________.
A. hair and eyes B. crazy laugh C. clothing
Part 2. Listen to the recording twice. Complete the form below. Write no more than two
words or numbers for each answer.
Example Answer
Purpose placing an advertisement.
Laptop FOR Sale
Condition: Almost new
Weight: (6) …………………………….
Make:Allegro
Memory: (7) …………………………….GB
Sceen: (8) …………………………….cm
Touch pad but with mouse
Number of ports: Two
Battery lasts: (9) …………………………….
Latest programmes: Not (10) …………………………….
Extras:
Web cam
Printer with:(11) ……………………….and …………………………
Smart case:
Price: (12) …………………………….
Contact details
Name: David (13) …………………………….
E-mail address: DIB_7791@hotmail.com
Mobile number: (14) …………………………….
Advert placed: (15) …………………………….
SECTION II: LEXICO – GRAMMAR (7.0 points)
Part 1: Questions from 16 to 36. (3.0 points - 0.15/ each)
Choose the best answer to each question.
16. You find it hard to _____ your temper if you think someone is making a fool out of you.
A. cool B. lose C. recover D. keep
17. Remember to _____ your shoes when you are in a Japanese house.
A. take on B. take off C. take up D. put on
18. I have _____ information about her situation to tell you.
A. too few B. so many C. so little D. too little
19. America, as well as Japan, England, and Germany_____.
A. are developed countries B. are developing countries
C. is a developed country D. is developing countries
20. It's no good _____ your father about your failure.
A. to tell B. tell C. telling D. told
21. I’m afraid I don’t _____ your view on this matter, but let’s not quarrel about it.
A. correspond B. equate C. accord D. share
22. He got an excellent grade in his examination_____ the fact that he had not worked
particularly hard.
A. on account of B. because C. in spite of D. although
23. He always _____ the crossword in the newspaper before breakfast.
A. writes B. makes C. works D. does
24. He just couldn't open the jar _____ hard he tried.
A. however B. whatever C. moreover D. even
25. “Today’s my 20th birthday.” - “______”
A. Take care! B. Many happy returns!
C. Have a good time! D. I don’t understand.
26. Tom: How are things going? Mary: _____.
A. I can't complain B. There are a lot of things
C. Not much D. That’s all right.
27. Vietnam's rice export this year will decrease ______ about 10%, compared with that of
last year.
A. with B. at C. by D. on
28. I’d like to contribute_____ the school Red Cross fund.
A. on B. to C. with D. for
29. “In spite of her deafness, she plays the violin very well.” Deafness means ______.
A. inability to speak B. inability to see
C. inability to hear D. mentally impairment
30. I am going to have my eye_____ tomorrow.
A. to test B. testC. testing D. tested
31. A: Which one do you like? B: I like ______ one of the two.
A. the bigger B. biggest C. bigger D. the biggest
32. Robert and his wife _____ to my house for tea yesterday evening.
A. came about B. came down C. came round D. came away
33. Each of the guests _____ a bunch of flowers.
A. is given B. are given C. give D. were given
34. We should arrive home safe and _____.
A. sound B. soundly C. warm D. warmly
35. The idea got a lot of _____ from the children’s parents.
A. opposition B. disagreement C. dislike D. denial
Part 2: Questions from 36 to 45. (2 points - 0.2/ each)
Use the correct form of the word in brackets to complete the following sentences.
36. …………………………is a part of life, but you have to be able to enjoy the others too. SAD
37. Without that, I wouldn't be happy even if I was the …………………………man in the RICH
world.
38. The music in the festival was so loud. It was …………………………! DEAF
39. I don't think my marks can get …………………………because I already study to the best of HIGH
my abilities.
40. The …………………………river fish in Europe lives in Spain! LARGE
41. Passeig de Gràcia is being …………………………so there is more room for people to walk WIDE
on the pavement.
42. I can …………………………that if you study you will have no problems passing this test. SURE
43. Nowadays there are lots of …………………………species of animals in the world, for DANGER
example, the panda.
44. I was …………………………to take the driving test by my friends. COURAGE
45. I see they've finally got round to …………………………the Shoreham road. WIDE
Part 3: Questions from 46 to 50.(1 point - 0.2/ each) Giaoan dethitienganh.info

Each of the following sentences has an error. Find the errors and correct them.
46. Nuclear powers production in the US is controlled by the Nuclear Regulatory Commission
[NRC].
47. All of we students must have an identification card in order to check books out of the
library.
48. A fiber-optic cable across the Pacific went into service in April 1989, link the United States and
Japan.
49. A good artist like a good engineer learns as much from their mistakes as from successes.
50. Almost all life depends to chemical reactions with oxygen to produce energy.
Part 4: Questions from 51 to 55.(1 point - 0.2/ each)
Fill in each of the sentences with correct prepositions.
51. He is always getting …………………trouble because of his carelessness.
52. You cross a cheque …………………drawing two lines in it and writing & co like this.
53. Not many old people approve …………………the ways of the younger people.
54. The local people look down …………………anyone who throws litter around their lovely park.
55. The editor never puts up …………………articles that they are boring.
SECTION III: READING (6.0 points)
Part 1. Questions 56 – 65 (2 points - 0.2/ each)
Read the passage below and choose the best answer to each question.
The ability to weep is a uniquely human form of emotional response. Some scientists
have suggested that human tears are (56) _____ of an aquatic past – but this does not seem
very likely. We cry from the moment we enter this world, for a number of reasons. Helpless
babies cry to persuade their parents that they are ill, hungry or uncomfortable. As they (57)
_____, they will also cry just to attract parental attention and will often stop when they get it.
The idea that having a good cry do you (58) _____ is a very old one and now it has
scientific validity since recent research into tears has shown that they (59) _____ a natural
painkiller called enkaphalin. By fighting sorrow and pain this chemical helps you feel better.
Weeping can increase the quantities of enkaphalin you (60) _____.
Unfortunately, in our society we impose restrictions upon this naturally (61) _____ activity.
Because some people still regard it as a (62) _____ of weakness in men, boys in particular are
admonished when they cry. This kind of repression can only increase stress, both emotionally
and physically.
Tears of emotion also help the body (63) _____ itself of toxic chemical waste, for there
is more protein in them than in tears resulting from cold winds or other irritants. Crying
comforts, calms and can be very enjoyable – (64) _____ the popularity of highly emotional
films which are commonly (65) _____ “weepies”. It seems that people enjoy crying together
almost as much as laughing together.
56. A. witness B. evidence C. result D. display
57. A. evolveB. change C. develop D. alter
58. A. better B. fineC. good D. well
59. A. contain B. retain C. hold D. keep
60. A. construct B. achieve C. provide D. produce
61. A. curing B. treating C. healing D. improving
62. A. hint B. symbol C. feature D. sign
63. A. release B. rid C. loosen D. expel
64. A. consider B. remark C. distinguish D. regard
65. A. named B. entitled C. subtitled D. called
Part 2. Questions 66 – 75 (2.0 points - 0.2/ each)
Read the passage below and choose the best answer to each question
By the mid-nineteenth century, the term "icebox" had entered the American language,
but ice was still only beginning to affect the diet of ordinary citizens in the United States. The
ice trade grew with the growth of cities. Ice was used in hotels, taverns, and hospitals, and by
some forward-looking city dealers in fresh meat, fresh fish, and butter. After the Civil
War(1861-1865), as ice was used to refrigerate freight cars, it also came into household use.
Even before 1880, half the ice sold in New York, Philadelphia, and Baltimore, and one-third of
that sold in Boston and Chicago, went to families for their own use. This had become possible
because a new household convenience, the icebox, a precursor of the modern refrigerator,
had been invented. Making an efficient ice box was not as easy as we might now suppose. In
the early nineteenth century, the knowledge of the physics of heat, which was essential to a
science of refrigeration, was rudimentary. The commonsense notion that the best icebox was
one that prevented the ice from melting was of course mistaken, for it was the melting of the
ice that performed the cooling. Nevertheless, early efforts to economize ice included
wrapping the ice in blankets, which kept the ice from doing its job. Not until near the end of
the nineteenth century did inventors achieve the delicate balance of insulation and circulation
needed for an efficient icebox.
But as early as 1803, an ingenious Maryland farmer, Thomas Moore, had been on the right
track. He owned a farm about twenty miles outside the city of Washington, forwhich the
village of Georgetown was the market center. When he used an icebox of his own design to
transport his butter to market, he found that customers would pass up the rapidly melting
stuff in the tubs of his competitors to pay a premium price for his butter, still fresh and hard in
neat, one-pound bricks. One advantage of his icebox, Moore explained, was that farmers
would no longer have to travel to market at night in order to keep their produce cool.

66. What does the passage mainly discuss?


A. The influence of ice on the diet B. The development of refrigeration
C. The transportation of goods to market D. Sources of ice in the 19th century
67. According to the passage, when did the word "icebox" become part of the language of the
United States?
A. In 1803 B. Sometime before 1850
C. During the Civil War D. Near the end of the nineteenth century
68. The phrase "forward-looking" in line 3 is closest in meaning to ________.
A. progressive B. popular C. thrifty D. well-established
69. The author mentions fish in line 5 because ________.
A. many fish dealers also sold ice
B. fish was shipped in refrigerated freight cars
C. fish dealers were among the early commercial users of ice
D. fish was not part of the ordinary person's diet before the invention of the icebox
70. The word "it" in line 5 refers to ________.
A. fresh meat B. the Civil War C. ice D. a refrigerator
71. According to the passage, which of the following was an obstacle to the development of
the icebox?
A. Competition among the owners of refrigerated freight cars
B. The lack of a network for the distribution of ice
C. The use of insufficient insulation
D. Inadequate understanding of physics
72. The word "rudimentary" in line 10 is closest in meaning to ________.
A. growing B. undeveloped C. necessary D. uninteresting
73. According to the information in the second paragraph, an ideal icebox would ________.
A. completely prevent ice from melting B. stop air from circulating
C. allow ice to melt slowly D. use blankets to conserve ice
74. The author describes Thomas Moore as having been "on the right track" to indicate that
________.
A. the road to the market passed close to Moore's farm
B. Moore was an honest merchant
C. Moore was a prosperous farmer
D. Moore's design was fairly successful
75. According to the passage, Moore's icebox allowed him to ________.
A. charge more for his butter B. travel to market at night
C. manufacture butter more quickly D. produce ice all year round
Part 3: Questions 76 – 85 (2.0 points - 0.2/ each)
Read the text and fill in one word which best fits each gap.
THE BIRTH OF THE T-SHIRT
The T-shirt, or at least the T-shirt as we know it, was born in the theatre. When
Tennessee William's play A Streetcar Named Oesire opened in New York in December 1947, a
young actor (76) ………………… Marlon Brando went (77) …………………stage wearing a (78) …………………of blue
jeans and a bright, white, capped-sleeve T-shirt. It was the first time the T-shirt had (79)
…………………seen publicly as anything except an item of underwear and it set a fashion trend (80)

…………………was to last through until the end of the century. The idea for the T-shirt came from
Brando himself. He had worn one at rehearsals for the play. The director was (81) so
impressed by the look that was created that he asked Brando to wear the shirt in the play
itself. Brando may have seen the shirt being advertised (82) …………………the American company
Sears Roebuck. They had decided to market the shirt as a fashionable garment in its own
right, rather (83) …………………just something to be worn (84) …………………warmth beneath a denim
workshirt (85) …………………an army uniform. It was Brando. However, who popularized it, especially
with the release of the film version of Streetcar in 1951. A short leather jacket completed the
look that was to be adopted by teenage rebels in many countries for decades afterwards.
SECTION IV: WRITING (4.0 points)
Part 1: Questions from 86 to 90.(2 point- 0.4/ each)
Finish each of the following sentences in such a way that it means the same as the sentence
printed before it.
86. I'm sure they didn't plan this project.
They can't
87. People say he was dismissed last week.
He
88. "I'm sorry I missed the appointment with the doctor this morning." he said.
He apologised
89. Why don't you do your homework before going to class?
If I
90. This motorbike is cheaper than I thought I would be.
This motorbike is not
Part 2: Questions from 91 to 92.(2 points - 0.4/ each)
Complete the second sentence so that it has a similar meaning to the first sentence, using
the word given. Do not change the word given.
91. Without the help that Joe gave me, I don't think I would have finished the course. been
If it ……………………………………….………………….., I don't think I would have finished the course.
92. Susan doesn't intend to climb that mountain again. has
Susan ……………………………………….………………….. that mountain again.
93. After a long hard journey, I cheered up when I saw my home again. sight
After a long hard journey, I cheered up ……………………………………….………………….. of my home again.
94. I promised him that the situation would not be repeated in the future. word
I ……………………………………….…………… that there would be no repetition of the situation in the future.
95. Dario was just about to leave for the theatre when the phone rang. point
Dario ……………………………………….………………….. for the theatre when the phone rang.
……………………THE END ……………………...
- Thí sinh không được sử dụng tài liệu
- Giám thị không giải thích gì thêm

You might also like